Atrophic Plaques on the Back

Article Type
Changed
Thu, 01/10/2019 - 13:18
Display Headline
Atrophic Plaques on the Back

The Diagnosis: Atrophic Pityriasis Versicolor

Pityriasis versicolor lesions accompanied by skin atrophy were first reported by De Graciansky and Mery1 in 1971. Since then, few reports have been described and it remains a rare condition.2-5 It manifests with oval to round, ivory-colored lesions with a typically depressed and sometimes finely pleated surface.3 The pathogenesis of the skin atrophy remains controversial. In some of the cases described, the onset of atrophy was related to long-term use of topical steroids.1 This fact as well as impaired barrier function due to fungal infection may explain the atrophy occurring only in the pityriasis versicolor lesions.2,6,7 Some authors call this disease “pityriasis versicolor pseudoatrophicans.”7 However, case reports have been described without use of topical corticosteroids. Crowson and Magro8 maintained that skin atrophy in these cases may occur due to mechanisms of delayed-type hypersensitivity and coined the term atrophying pityriasis versicolor as a variant of this disease. Our patient did not report prior use of topical corticosteroids.

The differential diagnosis consists of other diseases that cause skin atrophy, such as collagen vascular diseases including anetoderma, morphea or atrophoderma, lupus erythematosus, dermatomyositis, and poikilodermatous T-cell dyscrasia; parapsoriasis or mycosis fungoides; sarcoidosis; cutis laxa; acrodermatitis chronica atrophicans; necrobiosis lipoidica; and atrophy due to intralesional steroid therapy.2,3,6-8 Histologic examination helps to achieve proper diagnosis. In our patient, cutaneous biopsy showed the presence of multiple short hyphae and spores in the horny layer with hematoxylin and eosin as well as periodic acid–Schiff stains, with typical “spaghetti and meatballs” appearance. Partial atrophy of the epidermis was observed with flattening of the epidermic ridges. A comparison with the normal areas could not be made because the biopsy was taken from cutaneous lesions without areas of uninvolved skin (Figure).

Multiple short hyphae and spores in the horny layer visible with periodic acid–Schiff staining, with typical “spaghetti and meatballs” appearance (original magnification ×20).
Multiple short hyphae and spores in the horny layer visible with periodic acid–Schiff staining, with typical “spaghetti and meatballs” appearance (original magnification ×20).

Treatment of this variant does not differ from conventional therapies for pityriasis versicolor, except that a longer treatment period might be required. Atrophy usually disappears, showing that atrophic pityriasis versicolor has a relatively good prognosis compared with other diseases that cause skin atrophy.2

Our patient was treated with ketoconazole gel 2% once daily for 3 weeks with complete resolution of the lesions and no evidence of atrophy.

References
  1. De Graciansky P, Mery F. Atrophie sur pityriasis verscolor après corticotherapie locales prolongee. Bull Soc Fr Dermatol Syphiligr. 1971;78:295.
  2. Yang YS, Shin MK, Haw CR. Atrophying pityriasis versicolor: is this a new variant of pityriasis versicolor? Ann Dermatol. 2010;22:456-459.
  3. Romano C, Maritati E, Ghilardi A, et al. A case of pityriasis versicolor atrophicans. Mycoses. 2005;48:439-441.
  4. Park JS, Chae IS, Kim IY, et al. Achromatic atrophic macules and patches of upper extremities. Indian J Dermatol Venereol Leprol. 2013;79:270.
  5. Tellechea O, Cravo M, Brinca A, et al. Pityriasis versicolor atrophicans. Eur J Dermatol. 2012;22:287-288.
  6. Mazuecos Blanca J, García-Bravo B, Moreno Giménez JC, et al. Pseudoatrophic pityriasis versicolor. Med Cutan Ibero Lat Am. 1990;18:101-103.
  7. Tatnall FM, Rycroft RJ. Pityriasis versicolor with cutaneous atrophy induced by topical steroid application. Clin Exp Dermatol. 1985;10:258-261.
  8. Crowson AN, Magro CM. Atrophying tinea versicolor: a clinical and histological study of 12 patients. Int J Dermatol. 2003;42:928-932.
Article PDF
Author and Disclosure Information

From the Dermatology Department, Complexo Hospitalario de Pontevedra, Spain.

The authors report no conflict of interest.

Correspondence: Celia Posada, MD, Dermatology Department, Complexo Hospitalario de Pontevedra, Loureiro Crespo Nº 2. 36001 Pontevedra, Spain (cposada@aedv.es).

Issue
Cutis - 94(4)
Publications
Page Number
E7-E8
Legacy Keywords
atrophic pityriasis versicolor, skin atrophy
Sections
Author and Disclosure Information

From the Dermatology Department, Complexo Hospitalario de Pontevedra, Spain.

The authors report no conflict of interest.

Correspondence: Celia Posada, MD, Dermatology Department, Complexo Hospitalario de Pontevedra, Loureiro Crespo Nº 2. 36001 Pontevedra, Spain (cposada@aedv.es).

Author and Disclosure Information

From the Dermatology Department, Complexo Hospitalario de Pontevedra, Spain.

The authors report no conflict of interest.

Correspondence: Celia Posada, MD, Dermatology Department, Complexo Hospitalario de Pontevedra, Loureiro Crespo Nº 2. 36001 Pontevedra, Spain (cposada@aedv.es).

Article PDF
Article PDF

The Diagnosis: Atrophic Pityriasis Versicolor

Pityriasis versicolor lesions accompanied by skin atrophy were first reported by De Graciansky and Mery1 in 1971. Since then, few reports have been described and it remains a rare condition.2-5 It manifests with oval to round, ivory-colored lesions with a typically depressed and sometimes finely pleated surface.3 The pathogenesis of the skin atrophy remains controversial. In some of the cases described, the onset of atrophy was related to long-term use of topical steroids.1 This fact as well as impaired barrier function due to fungal infection may explain the atrophy occurring only in the pityriasis versicolor lesions.2,6,7 Some authors call this disease “pityriasis versicolor pseudoatrophicans.”7 However, case reports have been described without use of topical corticosteroids. Crowson and Magro8 maintained that skin atrophy in these cases may occur due to mechanisms of delayed-type hypersensitivity and coined the term atrophying pityriasis versicolor as a variant of this disease. Our patient did not report prior use of topical corticosteroids.

The differential diagnosis consists of other diseases that cause skin atrophy, such as collagen vascular diseases including anetoderma, morphea or atrophoderma, lupus erythematosus, dermatomyositis, and poikilodermatous T-cell dyscrasia; parapsoriasis or mycosis fungoides; sarcoidosis; cutis laxa; acrodermatitis chronica atrophicans; necrobiosis lipoidica; and atrophy due to intralesional steroid therapy.2,3,6-8 Histologic examination helps to achieve proper diagnosis. In our patient, cutaneous biopsy showed the presence of multiple short hyphae and spores in the horny layer with hematoxylin and eosin as well as periodic acid–Schiff stains, with typical “spaghetti and meatballs” appearance. Partial atrophy of the epidermis was observed with flattening of the epidermic ridges. A comparison with the normal areas could not be made because the biopsy was taken from cutaneous lesions without areas of uninvolved skin (Figure).

Multiple short hyphae and spores in the horny layer visible with periodic acid–Schiff staining, with typical “spaghetti and meatballs” appearance (original magnification ×20).
Multiple short hyphae and spores in the horny layer visible with periodic acid–Schiff staining, with typical “spaghetti and meatballs” appearance (original magnification ×20).

Treatment of this variant does not differ from conventional therapies for pityriasis versicolor, except that a longer treatment period might be required. Atrophy usually disappears, showing that atrophic pityriasis versicolor has a relatively good prognosis compared with other diseases that cause skin atrophy.2

Our patient was treated with ketoconazole gel 2% once daily for 3 weeks with complete resolution of the lesions and no evidence of atrophy.

The Diagnosis: Atrophic Pityriasis Versicolor

Pityriasis versicolor lesions accompanied by skin atrophy were first reported by De Graciansky and Mery1 in 1971. Since then, few reports have been described and it remains a rare condition.2-5 It manifests with oval to round, ivory-colored lesions with a typically depressed and sometimes finely pleated surface.3 The pathogenesis of the skin atrophy remains controversial. In some of the cases described, the onset of atrophy was related to long-term use of topical steroids.1 This fact as well as impaired barrier function due to fungal infection may explain the atrophy occurring only in the pityriasis versicolor lesions.2,6,7 Some authors call this disease “pityriasis versicolor pseudoatrophicans.”7 However, case reports have been described without use of topical corticosteroids. Crowson and Magro8 maintained that skin atrophy in these cases may occur due to mechanisms of delayed-type hypersensitivity and coined the term atrophying pityriasis versicolor as a variant of this disease. Our patient did not report prior use of topical corticosteroids.

The differential diagnosis consists of other diseases that cause skin atrophy, such as collagen vascular diseases including anetoderma, morphea or atrophoderma, lupus erythematosus, dermatomyositis, and poikilodermatous T-cell dyscrasia; parapsoriasis or mycosis fungoides; sarcoidosis; cutis laxa; acrodermatitis chronica atrophicans; necrobiosis lipoidica; and atrophy due to intralesional steroid therapy.2,3,6-8 Histologic examination helps to achieve proper diagnosis. In our patient, cutaneous biopsy showed the presence of multiple short hyphae and spores in the horny layer with hematoxylin and eosin as well as periodic acid–Schiff stains, with typical “spaghetti and meatballs” appearance. Partial atrophy of the epidermis was observed with flattening of the epidermic ridges. A comparison with the normal areas could not be made because the biopsy was taken from cutaneous lesions without areas of uninvolved skin (Figure).

Multiple short hyphae and spores in the horny layer visible with periodic acid–Schiff staining, with typical “spaghetti and meatballs” appearance (original magnification ×20).
Multiple short hyphae and spores in the horny layer visible with periodic acid–Schiff staining, with typical “spaghetti and meatballs” appearance (original magnification ×20).

Treatment of this variant does not differ from conventional therapies for pityriasis versicolor, except that a longer treatment period might be required. Atrophy usually disappears, showing that atrophic pityriasis versicolor has a relatively good prognosis compared with other diseases that cause skin atrophy.2

Our patient was treated with ketoconazole gel 2% once daily for 3 weeks with complete resolution of the lesions and no evidence of atrophy.

References
  1. De Graciansky P, Mery F. Atrophie sur pityriasis verscolor après corticotherapie locales prolongee. Bull Soc Fr Dermatol Syphiligr. 1971;78:295.
  2. Yang YS, Shin MK, Haw CR. Atrophying pityriasis versicolor: is this a new variant of pityriasis versicolor? Ann Dermatol. 2010;22:456-459.
  3. Romano C, Maritati E, Ghilardi A, et al. A case of pityriasis versicolor atrophicans. Mycoses. 2005;48:439-441.
  4. Park JS, Chae IS, Kim IY, et al. Achromatic atrophic macules and patches of upper extremities. Indian J Dermatol Venereol Leprol. 2013;79:270.
  5. Tellechea O, Cravo M, Brinca A, et al. Pityriasis versicolor atrophicans. Eur J Dermatol. 2012;22:287-288.
  6. Mazuecos Blanca J, García-Bravo B, Moreno Giménez JC, et al. Pseudoatrophic pityriasis versicolor. Med Cutan Ibero Lat Am. 1990;18:101-103.
  7. Tatnall FM, Rycroft RJ. Pityriasis versicolor with cutaneous atrophy induced by topical steroid application. Clin Exp Dermatol. 1985;10:258-261.
  8. Crowson AN, Magro CM. Atrophying tinea versicolor: a clinical and histological study of 12 patients. Int J Dermatol. 2003;42:928-932.
References
  1. De Graciansky P, Mery F. Atrophie sur pityriasis verscolor après corticotherapie locales prolongee. Bull Soc Fr Dermatol Syphiligr. 1971;78:295.
  2. Yang YS, Shin MK, Haw CR. Atrophying pityriasis versicolor: is this a new variant of pityriasis versicolor? Ann Dermatol. 2010;22:456-459.
  3. Romano C, Maritati E, Ghilardi A, et al. A case of pityriasis versicolor atrophicans. Mycoses. 2005;48:439-441.
  4. Park JS, Chae IS, Kim IY, et al. Achromatic atrophic macules and patches of upper extremities. Indian J Dermatol Venereol Leprol. 2013;79:270.
  5. Tellechea O, Cravo M, Brinca A, et al. Pityriasis versicolor atrophicans. Eur J Dermatol. 2012;22:287-288.
  6. Mazuecos Blanca J, García-Bravo B, Moreno Giménez JC, et al. Pseudoatrophic pityriasis versicolor. Med Cutan Ibero Lat Am. 1990;18:101-103.
  7. Tatnall FM, Rycroft RJ. Pityriasis versicolor with cutaneous atrophy induced by topical steroid application. Clin Exp Dermatol. 1985;10:258-261.
  8. Crowson AN, Magro CM. Atrophying tinea versicolor: a clinical and histological study of 12 patients. Int J Dermatol. 2003;42:928-932.
Issue
Cutis - 94(4)
Issue
Cutis - 94(4)
Page Number
E7-E8
Page Number
E7-E8
Publications
Publications
Article Type
Display Headline
Atrophic Plaques on the Back
Display Headline
Atrophic Plaques on the Back
Legacy Keywords
atrophic pityriasis versicolor, skin atrophy
Legacy Keywords
atrophic pityriasis versicolor, skin atrophy
Sections
Questionnaire Body

A 41-year-old woman presented with recurrent skin color and violaceous atrophic plaques that were slightly depressed and symmetrically distributed on the back and upper extremities. She had been given oral azithromycin for 3 days without improvement. Laboratory tests, including IgE levels, were within reference range. Her medical history was unremarkable.

 

Disallow All Ads
Alternative CME
Article PDF Media

Erythematous Seropurulent Ulcerations

Article Type
Changed
Thu, 01/10/2019 - 13:18
Display Headline
Erythematous Seropurulent Ulcerations

The Diagnosis: Cutaneous Leishmaniasis

On examination, the patient had multiple punched-out ulcers with indurated borders and surrounding erythema arranged in a sporotrichoid pattern from the left forearm to the left lateral chest (Figure 1).

 

Sporotrichoid spread of ulcers from the distal forearm to the lateral chest.
Figure 1. Sporotrichoid spread of ulcers from the distal forearm to the lateral chest.

Bacterial culture of a tissue specimen was negative, and tissue fungal culture failed to grow any organisms. Serological studies included a complete blood cell count with differential, a chemistry panel, and liver function tests, which were all unremarkable. Coccidioidomycosis and human immunodefi-ciency virus antibodies were negative. A 4-mm punch biopsy was obtained and sent to the Armed Forces Institute of Pathology for review. Histopathologic examination revealed marked inflammation with ill-formed noncaseating granulomas and focal ulceration, necrosis in the deep dermis, and both intra-cellular and extracellular amastigotes within areas of necrosis (Figures 2 and 3).

 

Mixed dermal inflammation with ill-formed, noncaseating granulomas (H&E, original magnification ×100).
Figure 2. Mixed dermal inflammation with ill-formed, noncaseating granulomas (H&E, original magnification ×100).

 

 

Multiple intracellular amastigotes are highlighted by the special stain, appearing as tiny “granules” in the cytoplasm of histiocytes (Giemsa, original magnification ×400).
Figure 3. Multiple intracellular amastigotes are highlighted by the special stain, appearing as tiny “granules” in the cytoplasm of histiocytes (Giemsa, original magnification ×400).

The rise in the number of cases of cutaneous leishmaniasis in the United States, particularly in the veteran population, can be attributed to the recent conflicts in the Middle East and Afghanistan. Infection with Leishmania species can result in a variety of clinical presentations, ranging from localized, self-limited cutaneous lesions to a life-threatening infection with visceral involvement.1 Additionally, the host immune response is variable. This variation in clinical presentation and disease progression explains why there is no single best treatment identified for leishmaniasis to date.

The clinical pattern of spread along the lymphatics in this patient is unique. The differential diagnosis of lesions with sporotrichoid spread includes Mycobacterium marinum and other atypical mycobacterial infections, Sporothrix schenckii, nocardiosis, leishmaniasis, coccidioidomycosis, tularemia, cat scratch disease, anthrax, chromoblastomycosis, pyogenic bacteria, and other fungal or bacterial infections. With such a broad differential diagnosis, histologic confirmation is paramount.

The most widely used pharmacotherapy for leishmaniasis is with pentavalent antimony compounds, which have been studied in randomized controlled trials for leishmaniasis more than any other drug.2 These antimony compounds are associated with a large spectrum of clinical adverse events, and there is increasing evidence for emerging parasite resistance to the antimonies.3-5 Historically, amphotericin B was considered a second-line treatment of leishmaniasis due to its systemic toxicity.6 However, this treatment has come back into favor due to its newer, more tolerable, lipid-associated formulation.

Our patient was treated with intravenous liposomal amphotericin B at a dosage of 3 mg/kg daily for days 1 to 5, then again on days 14 and 21. He tolerated the therapeutic regimen without difficulty or adverse effects. The ulcers eventually became smaller and ceased to weep, fully healing over a course of several months.

References

 

1. Martin-Ezquerra G, Fisa R, Riera C, et al. Role of Leishmania spp. infestation in nondiagnostic cutaneous granulomatous lesions: report of a series of patients from a Western Mediterranean area. Br J Dermatol. 2009;161:320-325.

2. Khatami A, Firooz A, Gorouhi F, et al. Treatment of acute old world cutaneous leishmaniasis: a systemic review of the randomized controlled trials. J Am Acad Dermatol. 2007;57:335.e1-335.e29.

3. Rojas R, Valderrama L, Valderrama M, et al. Resistance to antimony and treatment failure in human Leishmania (Viannia) infection. J Infect Dis. 2006;193:1375-1383.

4. Hadighi R, Mohebali M, Boucher P, et al. Unresponsiveness to glucantime treatment in Iranian cutaneous leishmaniasis due to drug-resistant Leishmania tropica parasites. PLoS Med. 2006;3:e162.

5. Croft SL, Sundar S, Fairlamb AH. Drug resistance in leishmaniasis. Clin Microbiol Rev. 2006;19:111-126.

6. Croft S, Seifert K, Yardley V. Current scenario of drug development for leishmaniasis. Indian J Med Res. 2006;123:399-410.

Article PDF
Author and Disclosure Information

Drs. Nino and Lee are from the Department of Dermatology, Loma Linda University, California. Drs. Macknet, Hirokane, and Damodaran are from the Department of Dermatology, Veterans Affairs Lorma Linda Healthcare System.

The authors report no conflict of interest.

Correspondence: Tanya Nino, MD, Loma Linda University Department of Dermatology, 11370 Anderson St, Ste 2600, Loma Linda, CA 92354 (tanyanino@gmail.com).

Issue
Cutis - 94(4)
Publications
Topics
Page Number
E4-E6
Legacy Keywords
ulcerated skin lesions, sporotrichoid pattern, Erythematous Seropurulent Ulcerations
Sections
Author and Disclosure Information

Drs. Nino and Lee are from the Department of Dermatology, Loma Linda University, California. Drs. Macknet, Hirokane, and Damodaran are from the Department of Dermatology, Veterans Affairs Lorma Linda Healthcare System.

The authors report no conflict of interest.

Correspondence: Tanya Nino, MD, Loma Linda University Department of Dermatology, 11370 Anderson St, Ste 2600, Loma Linda, CA 92354 (tanyanino@gmail.com).

Author and Disclosure Information

Drs. Nino and Lee are from the Department of Dermatology, Loma Linda University, California. Drs. Macknet, Hirokane, and Damodaran are from the Department of Dermatology, Veterans Affairs Lorma Linda Healthcare System.

The authors report no conflict of interest.

Correspondence: Tanya Nino, MD, Loma Linda University Department of Dermatology, 11370 Anderson St, Ste 2600, Loma Linda, CA 92354 (tanyanino@gmail.com).

Article PDF
Article PDF

The Diagnosis: Cutaneous Leishmaniasis

On examination, the patient had multiple punched-out ulcers with indurated borders and surrounding erythema arranged in a sporotrichoid pattern from the left forearm to the left lateral chest (Figure 1).

 

Sporotrichoid spread of ulcers from the distal forearm to the lateral chest.
Figure 1. Sporotrichoid spread of ulcers from the distal forearm to the lateral chest.

Bacterial culture of a tissue specimen was negative, and tissue fungal culture failed to grow any organisms. Serological studies included a complete blood cell count with differential, a chemistry panel, and liver function tests, which were all unremarkable. Coccidioidomycosis and human immunodefi-ciency virus antibodies were negative. A 4-mm punch biopsy was obtained and sent to the Armed Forces Institute of Pathology for review. Histopathologic examination revealed marked inflammation with ill-formed noncaseating granulomas and focal ulceration, necrosis in the deep dermis, and both intra-cellular and extracellular amastigotes within areas of necrosis (Figures 2 and 3).

 

Mixed dermal inflammation with ill-formed, noncaseating granulomas (H&E, original magnification ×100).
Figure 2. Mixed dermal inflammation with ill-formed, noncaseating granulomas (H&E, original magnification ×100).

 

 

Multiple intracellular amastigotes are highlighted by the special stain, appearing as tiny “granules” in the cytoplasm of histiocytes (Giemsa, original magnification ×400).
Figure 3. Multiple intracellular amastigotes are highlighted by the special stain, appearing as tiny “granules” in the cytoplasm of histiocytes (Giemsa, original magnification ×400).

The rise in the number of cases of cutaneous leishmaniasis in the United States, particularly in the veteran population, can be attributed to the recent conflicts in the Middle East and Afghanistan. Infection with Leishmania species can result in a variety of clinical presentations, ranging from localized, self-limited cutaneous lesions to a life-threatening infection with visceral involvement.1 Additionally, the host immune response is variable. This variation in clinical presentation and disease progression explains why there is no single best treatment identified for leishmaniasis to date.

The clinical pattern of spread along the lymphatics in this patient is unique. The differential diagnosis of lesions with sporotrichoid spread includes Mycobacterium marinum and other atypical mycobacterial infections, Sporothrix schenckii, nocardiosis, leishmaniasis, coccidioidomycosis, tularemia, cat scratch disease, anthrax, chromoblastomycosis, pyogenic bacteria, and other fungal or bacterial infections. With such a broad differential diagnosis, histologic confirmation is paramount.

The most widely used pharmacotherapy for leishmaniasis is with pentavalent antimony compounds, which have been studied in randomized controlled trials for leishmaniasis more than any other drug.2 These antimony compounds are associated with a large spectrum of clinical adverse events, and there is increasing evidence for emerging parasite resistance to the antimonies.3-5 Historically, amphotericin B was considered a second-line treatment of leishmaniasis due to its systemic toxicity.6 However, this treatment has come back into favor due to its newer, more tolerable, lipid-associated formulation.

Our patient was treated with intravenous liposomal amphotericin B at a dosage of 3 mg/kg daily for days 1 to 5, then again on days 14 and 21. He tolerated the therapeutic regimen without difficulty or adverse effects. The ulcers eventually became smaller and ceased to weep, fully healing over a course of several months.

The Diagnosis: Cutaneous Leishmaniasis

On examination, the patient had multiple punched-out ulcers with indurated borders and surrounding erythema arranged in a sporotrichoid pattern from the left forearm to the left lateral chest (Figure 1).

 

Sporotrichoid spread of ulcers from the distal forearm to the lateral chest.
Figure 1. Sporotrichoid spread of ulcers from the distal forearm to the lateral chest.

Bacterial culture of a tissue specimen was negative, and tissue fungal culture failed to grow any organisms. Serological studies included a complete blood cell count with differential, a chemistry panel, and liver function tests, which were all unremarkable. Coccidioidomycosis and human immunodefi-ciency virus antibodies were negative. A 4-mm punch biopsy was obtained and sent to the Armed Forces Institute of Pathology for review. Histopathologic examination revealed marked inflammation with ill-formed noncaseating granulomas and focal ulceration, necrosis in the deep dermis, and both intra-cellular and extracellular amastigotes within areas of necrosis (Figures 2 and 3).

 

Mixed dermal inflammation with ill-formed, noncaseating granulomas (H&E, original magnification ×100).
Figure 2. Mixed dermal inflammation with ill-formed, noncaseating granulomas (H&E, original magnification ×100).

 

 

Multiple intracellular amastigotes are highlighted by the special stain, appearing as tiny “granules” in the cytoplasm of histiocytes (Giemsa, original magnification ×400).
Figure 3. Multiple intracellular amastigotes are highlighted by the special stain, appearing as tiny “granules” in the cytoplasm of histiocytes (Giemsa, original magnification ×400).

The rise in the number of cases of cutaneous leishmaniasis in the United States, particularly in the veteran population, can be attributed to the recent conflicts in the Middle East and Afghanistan. Infection with Leishmania species can result in a variety of clinical presentations, ranging from localized, self-limited cutaneous lesions to a life-threatening infection with visceral involvement.1 Additionally, the host immune response is variable. This variation in clinical presentation and disease progression explains why there is no single best treatment identified for leishmaniasis to date.

The clinical pattern of spread along the lymphatics in this patient is unique. The differential diagnosis of lesions with sporotrichoid spread includes Mycobacterium marinum and other atypical mycobacterial infections, Sporothrix schenckii, nocardiosis, leishmaniasis, coccidioidomycosis, tularemia, cat scratch disease, anthrax, chromoblastomycosis, pyogenic bacteria, and other fungal or bacterial infections. With such a broad differential diagnosis, histologic confirmation is paramount.

The most widely used pharmacotherapy for leishmaniasis is with pentavalent antimony compounds, which have been studied in randomized controlled trials for leishmaniasis more than any other drug.2 These antimony compounds are associated with a large spectrum of clinical adverse events, and there is increasing evidence for emerging parasite resistance to the antimonies.3-5 Historically, amphotericin B was considered a second-line treatment of leishmaniasis due to its systemic toxicity.6 However, this treatment has come back into favor due to its newer, more tolerable, lipid-associated formulation.

Our patient was treated with intravenous liposomal amphotericin B at a dosage of 3 mg/kg daily for days 1 to 5, then again on days 14 and 21. He tolerated the therapeutic regimen without difficulty or adverse effects. The ulcers eventually became smaller and ceased to weep, fully healing over a course of several months.

References

 

1. Martin-Ezquerra G, Fisa R, Riera C, et al. Role of Leishmania spp. infestation in nondiagnostic cutaneous granulomatous lesions: report of a series of patients from a Western Mediterranean area. Br J Dermatol. 2009;161:320-325.

2. Khatami A, Firooz A, Gorouhi F, et al. Treatment of acute old world cutaneous leishmaniasis: a systemic review of the randomized controlled trials. J Am Acad Dermatol. 2007;57:335.e1-335.e29.

3. Rojas R, Valderrama L, Valderrama M, et al. Resistance to antimony and treatment failure in human Leishmania (Viannia) infection. J Infect Dis. 2006;193:1375-1383.

4. Hadighi R, Mohebali M, Boucher P, et al. Unresponsiveness to glucantime treatment in Iranian cutaneous leishmaniasis due to drug-resistant Leishmania tropica parasites. PLoS Med. 2006;3:e162.

5. Croft SL, Sundar S, Fairlamb AH. Drug resistance in leishmaniasis. Clin Microbiol Rev. 2006;19:111-126.

6. Croft S, Seifert K, Yardley V. Current scenario of drug development for leishmaniasis. Indian J Med Res. 2006;123:399-410.

References

 

1. Martin-Ezquerra G, Fisa R, Riera C, et al. Role of Leishmania spp. infestation in nondiagnostic cutaneous granulomatous lesions: report of a series of patients from a Western Mediterranean area. Br J Dermatol. 2009;161:320-325.

2. Khatami A, Firooz A, Gorouhi F, et al. Treatment of acute old world cutaneous leishmaniasis: a systemic review of the randomized controlled trials. J Am Acad Dermatol. 2007;57:335.e1-335.e29.

3. Rojas R, Valderrama L, Valderrama M, et al. Resistance to antimony and treatment failure in human Leishmania (Viannia) infection. J Infect Dis. 2006;193:1375-1383.

4. Hadighi R, Mohebali M, Boucher P, et al. Unresponsiveness to glucantime treatment in Iranian cutaneous leishmaniasis due to drug-resistant Leishmania tropica parasites. PLoS Med. 2006;3:e162.

5. Croft SL, Sundar S, Fairlamb AH. Drug resistance in leishmaniasis. Clin Microbiol Rev. 2006;19:111-126.

6. Croft S, Seifert K, Yardley V. Current scenario of drug development for leishmaniasis. Indian J Med Res. 2006;123:399-410.

Issue
Cutis - 94(4)
Issue
Cutis - 94(4)
Page Number
E4-E6
Page Number
E4-E6
Publications
Publications
Topics
Article Type
Display Headline
Erythematous Seropurulent Ulcerations
Display Headline
Erythematous Seropurulent Ulcerations
Legacy Keywords
ulcerated skin lesions, sporotrichoid pattern, Erythematous Seropurulent Ulcerations
Legacy Keywords
ulcerated skin lesions, sporotrichoid pattern, Erythematous Seropurulent Ulcerations
Sections
Questionnaire Body

A 34-year-old male veteran who was otherwise healthy presented with multiple ulcerated skin lesions on the left arm and forearm as well as the chest. After returning to the United States from being stationed in Qatar and Saudi Arabia, he noticed multiple “bug bites” on the left arm that eventually progressed to larger crusted ulcerations. He denied fever, chills, nausea, vomiting, abdominal pain, tenderness, or any other symptoms. He had been given doxycycline for a possible bacterial infection, but the lesions did not improve.

 

Disallow All Ads
Alternative CME
Article PDF Media

Asymptomatic Annular Plaques on the Neck

Article Type
Changed
Thu, 01/10/2019 - 13:18
Display Headline
Asymptomatic Annular Plaques on the Neck

The Diagnosis: D-Penicillamine–Induced Elastosis Perforans Serpiginosa

A 27-year-old woman was referred to our clinic by her rheumatologist with a 2×5-cm annular plaque on the right side of the lateral neck along with nummular plaques on the left side of 1 year’s duration (Figure 1). Her medical history was notable for systemic sclerosis that had been treated with oral D-penicillamine (750 mg daily) for the last 10 years. Histopathologic examination of the skin biopsy specimen revealed an epidermis with perforating channels of elastic fibers admixed with collagen (Figure 2). Verhoeff–van Gieson elastin stain highlighted “bramble bush or lumpy-bumpy” appearance of elastic fibers in the dermis (Figure 3), which confirmed the diagnosis of D-penicillamine–induced elastosis perforans serpiginosa (EPS).

Figure 1. Nummular plaques on the left side of the
lateral neck.

Figure 2. A punch biopsy specimen showed an epidermis with perforating channels of elastic fibers admixed with collagen (H&E, original magnification ×10).

Figure 3. Verhoeff–van Gieson elastin stain highlighted “bramble bush or lumpy-bumpy” appearance of elastic fibers in the dermis (original magnification ×40).

Elastosis perforans serpiginosa is a rare entity that may present in many different settings. Some associated genetic conditions include Down syndrome, pseudoxanthoma elasticum, Marfan syndrome, Ehlers-Danlos syndrome, acrogeria, osteogenesis imperfecta, Rothmund-Thomson syndrome, and moya-moya disease. Elastosis perforans serpiginosa also may be inherited in rare cases in an autosomal-dominant pattern.1 There are solitary reports of EPS in the setting of renal disease, morphea, and systemic sclerosis. Most cases of EPS are iatrogenically acquired. As first reported in 1973, long-term D-penicillamine therapy for Wilson disease has been classically associated with the rare development of EPS.2 Putative mechanisms include copper chelation by D-penicillamine in the setting of altered copper homeostasis in Wilson disease and subsequent inhibition of elastic fiber cross-linking by copper-dependent lysyl oxidase. Another proposed mechanism is the direct inhibition of collagen cross-linking by D-penicillamine resulting in abnormal elastic fiber maturation.3 Outside of the context of Wilson disease, D-penicillamine–induced EPS also has developed during the treatment of juvenile rheumatoid arthritis and cystinuria.4 Our patient withsystemic sclerosis also exemplifies the possibility of developing EPS from long-term D-penicillamine therapy even in the absence of coexisting Wilson disease.

Elastosis perforans serpiginosa lesions classically present as asymptomatic, serpiginously arranged, hyperkeratotic papules, nodules, and annular plaques in young adults and children. Lesions usually present on the neck, though other locations have been described. Histologically, transepidermal elimination of elastic fibers, degenerated keratinocytes, and collagen is seen in the background of a foreign-body reaction with  hematoxylin and eosin stain. Elastin stains show increased thickened elastic fibers in the dermis underlying the perforation. The histology of D-penicillamine–induced EPS is distinctive in that the elastic fibers are arranged in a bramble bush pattern with lateral buds.

The clinical course of D-penicillamine–induced EPS is variable, ranging from slow to no resolution after drug discontinuation, with residual scarring, atrophy, and concern for systemic elastosis. Adjunctive therapies include oral and topical retinoids, cryotherapy, imiquimod, and CO2 laser.5 For our patient, tazarotene gel 0.1% was recommended, but the patient became pregnant soon after the diagnosis was made. Despite being untreated, her lesions have remarkably improved during her pregnancy.

References

 

1. Langeveld-Wildschut EG, Toonstra J, van Vloten WA, et al. Familial elastosis perforans serpiginosa. Arch Dermatol. 1993;129:205-207.

2. Pass F, Goldfischer S, Sternlieb I, et al. Elastosis perforans serpiginosa during penicillamine therapy for Wilson disease. Arch Dermatol. 1973;108:713-715.

3. Deguti MM, Mucenic M, Cancado EL, et al. Elastosis perforans serpiginosa secondary to D-penicillamine treatment in a Wilson’s disease patient. Am J Gastroenterol. 2002;97:2153-2154.

4. Sahn EE, Maize JC, Garen PD, et al. D-penicillamine–induced elastosis perforans serpiginosa in a child with juvenile rheumatoid arthritis. report of a case and review of the literature. J Am Acad Dermatol. 1989;20:979-988.

5. Atzori L, Pinna AL, Pau M, et al. D-penicillamine elastosis perforans serpiginosa: description of two cases and review of the literature. Dermatol Online J. 2011;17:3.

Article PDF
Author and Disclosure Information

From the Department of Dermatology, University of Pittsburgh, Pennsylvania.

The authors report no conflict of interest.

Correspondence: Lisa Maria Grandinetti, MD, University of Pittsburgh, Department of Dermatology, 200 Lothrop St, Medical Arts Bldg, 5th Floor, Pittsburgh, PA 15213 (grandinettilm@upmc.edu).

Issue
Cutis - 94(4)
Publications
Page Number
E1-E3
Legacy Keywords
asymptomatic plaques, Elastosis perforans serpiginosa, D-penicillamine,
Sections
Author and Disclosure Information

From the Department of Dermatology, University of Pittsburgh, Pennsylvania.

The authors report no conflict of interest.

Correspondence: Lisa Maria Grandinetti, MD, University of Pittsburgh, Department of Dermatology, 200 Lothrop St, Medical Arts Bldg, 5th Floor, Pittsburgh, PA 15213 (grandinettilm@upmc.edu).

Author and Disclosure Information

From the Department of Dermatology, University of Pittsburgh, Pennsylvania.

The authors report no conflict of interest.

Correspondence: Lisa Maria Grandinetti, MD, University of Pittsburgh, Department of Dermatology, 200 Lothrop St, Medical Arts Bldg, 5th Floor, Pittsburgh, PA 15213 (grandinettilm@upmc.edu).

Article PDF
Article PDF

The Diagnosis: D-Penicillamine–Induced Elastosis Perforans Serpiginosa

A 27-year-old woman was referred to our clinic by her rheumatologist with a 2×5-cm annular plaque on the right side of the lateral neck along with nummular plaques on the left side of 1 year’s duration (Figure 1). Her medical history was notable for systemic sclerosis that had been treated with oral D-penicillamine (750 mg daily) for the last 10 years. Histopathologic examination of the skin biopsy specimen revealed an epidermis with perforating channels of elastic fibers admixed with collagen (Figure 2). Verhoeff–van Gieson elastin stain highlighted “bramble bush or lumpy-bumpy” appearance of elastic fibers in the dermis (Figure 3), which confirmed the diagnosis of D-penicillamine–induced elastosis perforans serpiginosa (EPS).

Figure 1. Nummular plaques on the left side of the
lateral neck.

Figure 2. A punch biopsy specimen showed an epidermis with perforating channels of elastic fibers admixed with collagen (H&E, original magnification ×10).

Figure 3. Verhoeff–van Gieson elastin stain highlighted “bramble bush or lumpy-bumpy” appearance of elastic fibers in the dermis (original magnification ×40).

Elastosis perforans serpiginosa is a rare entity that may present in many different settings. Some associated genetic conditions include Down syndrome, pseudoxanthoma elasticum, Marfan syndrome, Ehlers-Danlos syndrome, acrogeria, osteogenesis imperfecta, Rothmund-Thomson syndrome, and moya-moya disease. Elastosis perforans serpiginosa also may be inherited in rare cases in an autosomal-dominant pattern.1 There are solitary reports of EPS in the setting of renal disease, morphea, and systemic sclerosis. Most cases of EPS are iatrogenically acquired. As first reported in 1973, long-term D-penicillamine therapy for Wilson disease has been classically associated with the rare development of EPS.2 Putative mechanisms include copper chelation by D-penicillamine in the setting of altered copper homeostasis in Wilson disease and subsequent inhibition of elastic fiber cross-linking by copper-dependent lysyl oxidase. Another proposed mechanism is the direct inhibition of collagen cross-linking by D-penicillamine resulting in abnormal elastic fiber maturation.3 Outside of the context of Wilson disease, D-penicillamine–induced EPS also has developed during the treatment of juvenile rheumatoid arthritis and cystinuria.4 Our patient withsystemic sclerosis also exemplifies the possibility of developing EPS from long-term D-penicillamine therapy even in the absence of coexisting Wilson disease.

Elastosis perforans serpiginosa lesions classically present as asymptomatic, serpiginously arranged, hyperkeratotic papules, nodules, and annular plaques in young adults and children. Lesions usually present on the neck, though other locations have been described. Histologically, transepidermal elimination of elastic fibers, degenerated keratinocytes, and collagen is seen in the background of a foreign-body reaction with  hematoxylin and eosin stain. Elastin stains show increased thickened elastic fibers in the dermis underlying the perforation. The histology of D-penicillamine–induced EPS is distinctive in that the elastic fibers are arranged in a bramble bush pattern with lateral buds.

The clinical course of D-penicillamine–induced EPS is variable, ranging from slow to no resolution after drug discontinuation, with residual scarring, atrophy, and concern for systemic elastosis. Adjunctive therapies include oral and topical retinoids, cryotherapy, imiquimod, and CO2 laser.5 For our patient, tazarotene gel 0.1% was recommended, but the patient became pregnant soon after the diagnosis was made. Despite being untreated, her lesions have remarkably improved during her pregnancy.

The Diagnosis: D-Penicillamine–Induced Elastosis Perforans Serpiginosa

A 27-year-old woman was referred to our clinic by her rheumatologist with a 2×5-cm annular plaque on the right side of the lateral neck along with nummular plaques on the left side of 1 year’s duration (Figure 1). Her medical history was notable for systemic sclerosis that had been treated with oral D-penicillamine (750 mg daily) for the last 10 years. Histopathologic examination of the skin biopsy specimen revealed an epidermis with perforating channels of elastic fibers admixed with collagen (Figure 2). Verhoeff–van Gieson elastin stain highlighted “bramble bush or lumpy-bumpy” appearance of elastic fibers in the dermis (Figure 3), which confirmed the diagnosis of D-penicillamine–induced elastosis perforans serpiginosa (EPS).

Figure 1. Nummular plaques on the left side of the
lateral neck.

Figure 2. A punch biopsy specimen showed an epidermis with perforating channels of elastic fibers admixed with collagen (H&E, original magnification ×10).

Figure 3. Verhoeff–van Gieson elastin stain highlighted “bramble bush or lumpy-bumpy” appearance of elastic fibers in the dermis (original magnification ×40).

Elastosis perforans serpiginosa is a rare entity that may present in many different settings. Some associated genetic conditions include Down syndrome, pseudoxanthoma elasticum, Marfan syndrome, Ehlers-Danlos syndrome, acrogeria, osteogenesis imperfecta, Rothmund-Thomson syndrome, and moya-moya disease. Elastosis perforans serpiginosa also may be inherited in rare cases in an autosomal-dominant pattern.1 There are solitary reports of EPS in the setting of renal disease, morphea, and systemic sclerosis. Most cases of EPS are iatrogenically acquired. As first reported in 1973, long-term D-penicillamine therapy for Wilson disease has been classically associated with the rare development of EPS.2 Putative mechanisms include copper chelation by D-penicillamine in the setting of altered copper homeostasis in Wilson disease and subsequent inhibition of elastic fiber cross-linking by copper-dependent lysyl oxidase. Another proposed mechanism is the direct inhibition of collagen cross-linking by D-penicillamine resulting in abnormal elastic fiber maturation.3 Outside of the context of Wilson disease, D-penicillamine–induced EPS also has developed during the treatment of juvenile rheumatoid arthritis and cystinuria.4 Our patient withsystemic sclerosis also exemplifies the possibility of developing EPS from long-term D-penicillamine therapy even in the absence of coexisting Wilson disease.

Elastosis perforans serpiginosa lesions classically present as asymptomatic, serpiginously arranged, hyperkeratotic papules, nodules, and annular plaques in young adults and children. Lesions usually present on the neck, though other locations have been described. Histologically, transepidermal elimination of elastic fibers, degenerated keratinocytes, and collagen is seen in the background of a foreign-body reaction with  hematoxylin and eosin stain. Elastin stains show increased thickened elastic fibers in the dermis underlying the perforation. The histology of D-penicillamine–induced EPS is distinctive in that the elastic fibers are arranged in a bramble bush pattern with lateral buds.

The clinical course of D-penicillamine–induced EPS is variable, ranging from slow to no resolution after drug discontinuation, with residual scarring, atrophy, and concern for systemic elastosis. Adjunctive therapies include oral and topical retinoids, cryotherapy, imiquimod, and CO2 laser.5 For our patient, tazarotene gel 0.1% was recommended, but the patient became pregnant soon after the diagnosis was made. Despite being untreated, her lesions have remarkably improved during her pregnancy.

References

 

1. Langeveld-Wildschut EG, Toonstra J, van Vloten WA, et al. Familial elastosis perforans serpiginosa. Arch Dermatol. 1993;129:205-207.

2. Pass F, Goldfischer S, Sternlieb I, et al. Elastosis perforans serpiginosa during penicillamine therapy for Wilson disease. Arch Dermatol. 1973;108:713-715.

3. Deguti MM, Mucenic M, Cancado EL, et al. Elastosis perforans serpiginosa secondary to D-penicillamine treatment in a Wilson’s disease patient. Am J Gastroenterol. 2002;97:2153-2154.

4. Sahn EE, Maize JC, Garen PD, et al. D-penicillamine–induced elastosis perforans serpiginosa in a child with juvenile rheumatoid arthritis. report of a case and review of the literature. J Am Acad Dermatol. 1989;20:979-988.

5. Atzori L, Pinna AL, Pau M, et al. D-penicillamine elastosis perforans serpiginosa: description of two cases and review of the literature. Dermatol Online J. 2011;17:3.

References

 

1. Langeveld-Wildschut EG, Toonstra J, van Vloten WA, et al. Familial elastosis perforans serpiginosa. Arch Dermatol. 1993;129:205-207.

2. Pass F, Goldfischer S, Sternlieb I, et al. Elastosis perforans serpiginosa during penicillamine therapy for Wilson disease. Arch Dermatol. 1973;108:713-715.

3. Deguti MM, Mucenic M, Cancado EL, et al. Elastosis perforans serpiginosa secondary to D-penicillamine treatment in a Wilson’s disease patient. Am J Gastroenterol. 2002;97:2153-2154.

4. Sahn EE, Maize JC, Garen PD, et al. D-penicillamine–induced elastosis perforans serpiginosa in a child with juvenile rheumatoid arthritis. report of a case and review of the literature. J Am Acad Dermatol. 1989;20:979-988.

5. Atzori L, Pinna AL, Pau M, et al. D-penicillamine elastosis perforans serpiginosa: description of two cases and review of the literature. Dermatol Online J. 2011;17:3.

Issue
Cutis - 94(4)
Issue
Cutis - 94(4)
Page Number
E1-E3
Page Number
E1-E3
Publications
Publications
Article Type
Display Headline
Asymptomatic Annular Plaques on the Neck
Display Headline
Asymptomatic Annular Plaques on the Neck
Legacy Keywords
asymptomatic plaques, Elastosis perforans serpiginosa, D-penicillamine,
Legacy Keywords
asymptomatic plaques, Elastosis perforans serpiginosa, D-penicillamine,
Sections
Questionnaire Body

A 27-year-old woman was referred to our clinic with asymptomatic plaques on the neck of 1 year’s duration. The patient reported that the annular plaque started as individual nummular lesions that subsequently coalesced. Her medical history included systemic sclerosis since 11 years of age. The patient’s systemic sclerosis had involved the esophagus, small bowel, and heart, requiring placement of a defibrillator. She was initially treated with hydroxychloroquine and then was placed on D-penicillamine for the last 10 years. Examination of the right side of the lateral neck revealed a 2×5-cm atrophic annular plaque with raised borders, and several 1.5-cm scaly, atrophic, nummular plaques were noted on the left side of the lateral neck. Other cutaneous manifestations included diffuse calcinosis cutis, sclerodactyly, and Raynaud phenomenon. A 4-mm punch biopsy of a raised margin of the annular plaque was performed.

 

Disallow All Ads
Alternative CME
Article PDF Media

Postop Patient Reports “Wound Infection”

Article Type
Changed
Tue, 12/13/2016 - 12:08
Display Headline
Postop Patient Reports “Wound Infection”

ANSWER

The correct answer is an allergic reaction to a contactant, most likely the triple-antibiotic ointment (choice “d”).

Irritant reactions to tape adhesive (choice “a”) are extremely common. However, the resultant rash would have been confined to the linear areas where the tape touched his skin.

Dissolving sutures, such as those used in this case, can provoke a “suture granuloma”—essentially a foreign body reaction to the suture material (choice “b”). But this would have caused a focal area of swelling and redness, and very possibly a show of pus.

Postop wound infections (choice “c”) are also quite common. However, they would not manifest solely with itching in a papulovesicular rash surrounding the wound. Had infection developed, the redness would have been broad-based, with ill-defined margins, and the patient’s complaint would have been of pain, not itching. No vesicles would have been seen with bacterial infection.

DISCUSSION

This case illustrates the phenomenon of “treatment as problem,” in which the medication the patient applies becomes more problematic than the condition being addressed. Reactions to the neomycin in triple-antibiotic ointment are common but still provoke considerable worry on the part of patients and providers alike, especially when mistaken for “infection.”

This patient, like many, was dubious of the diagnosis, pointing out that he had used this same topical medication on many occasions without incident (though not recently). What he didn’t know is that it takes repeated exposure to a given allergen to develop T-memory cells that eventually begin to react. This same phenomenon is seen with poison ivy; patients will recall the ability, as a child, to practically wallow in poison ivy with impunity, making them doubtful about being allergic to it as an adult.

Neomycin, an aminoglycoside with a fairly wide spectrum of antibacterial activity, was first noted as a contact allergen in 1952. It is such a notorious offender that it was named Allergen of the Year in 2010 by the American Contact Dermatology Society.

For the past 20 years, 7% to 13% of patch tests surveyed were positive for neomycin. For reasons not entirely clear, Americans older than 60 are 150% more likely to experience a reaction to neomycin than are younger patients. (It could simply be that they’ve had more chances for exposure.)

In another interesting twist, the ointment vehicle appears to play a role. A reaction to this preparation is considerably more likely than to the same drug in other forms (eg, powders, solutions, creams). This is true of most medications, such as topical steroids, which are effectively self-occluded by this vehicle.

Persons with impaired barrier function, such as those with atopic dermatitis or whose skin has been prepped for surgery, appear to be at increased risk for these types of contact dermatoses.

Though there are other items in the differential, the configuration of the papulovesicular rash and the sole symptom of itching are essentially pathognomic for contact dermatitis. Besides the use of potent topical steroids for a few days, the real “cure” for this problem is for the patient to switch to “double-antibiotic” creams or ointments that do not include neomycin. 

Article PDF
Author and Disclosure Information

 

Joe R. Monroe, MPAS, PA, ­practices at Dawkins ­Dermatology Clinic in Oklahoma City. He is also the founder of the Society of ­Dermatology ­Physician ­Assistants.

Issue
Clinician Reviews - 24(10)
Publications
Topics
Page Number
16-17
Legacy Keywords
sharply defined, bright pink, papulovesicular rash, triple-antibiotic ointment, neomycin, contact allergen
Sections
Author and Disclosure Information

 

Joe R. Monroe, MPAS, PA, ­practices at Dawkins ­Dermatology Clinic in Oklahoma City. He is also the founder of the Society of ­Dermatology ­Physician ­Assistants.

Author and Disclosure Information

 

Joe R. Monroe, MPAS, PA, ­practices at Dawkins ­Dermatology Clinic in Oklahoma City. He is also the founder of the Society of ­Dermatology ­Physician ­Assistants.

Article PDF
Article PDF

ANSWER

The correct answer is an allergic reaction to a contactant, most likely the triple-antibiotic ointment (choice “d”).

Irritant reactions to tape adhesive (choice “a”) are extremely common. However, the resultant rash would have been confined to the linear areas where the tape touched his skin.

Dissolving sutures, such as those used in this case, can provoke a “suture granuloma”—essentially a foreign body reaction to the suture material (choice “b”). But this would have caused a focal area of swelling and redness, and very possibly a show of pus.

Postop wound infections (choice “c”) are also quite common. However, they would not manifest solely with itching in a papulovesicular rash surrounding the wound. Had infection developed, the redness would have been broad-based, with ill-defined margins, and the patient’s complaint would have been of pain, not itching. No vesicles would have been seen with bacterial infection.

DISCUSSION

This case illustrates the phenomenon of “treatment as problem,” in which the medication the patient applies becomes more problematic than the condition being addressed. Reactions to the neomycin in triple-antibiotic ointment are common but still provoke considerable worry on the part of patients and providers alike, especially when mistaken for “infection.”

This patient, like many, was dubious of the diagnosis, pointing out that he had used this same topical medication on many occasions without incident (though not recently). What he didn’t know is that it takes repeated exposure to a given allergen to develop T-memory cells that eventually begin to react. This same phenomenon is seen with poison ivy; patients will recall the ability, as a child, to practically wallow in poison ivy with impunity, making them doubtful about being allergic to it as an adult.

Neomycin, an aminoglycoside with a fairly wide spectrum of antibacterial activity, was first noted as a contact allergen in 1952. It is such a notorious offender that it was named Allergen of the Year in 2010 by the American Contact Dermatology Society.

For the past 20 years, 7% to 13% of patch tests surveyed were positive for neomycin. For reasons not entirely clear, Americans older than 60 are 150% more likely to experience a reaction to neomycin than are younger patients. (It could simply be that they’ve had more chances for exposure.)

In another interesting twist, the ointment vehicle appears to play a role. A reaction to this preparation is considerably more likely than to the same drug in other forms (eg, powders, solutions, creams). This is true of most medications, such as topical steroids, which are effectively self-occluded by this vehicle.

Persons with impaired barrier function, such as those with atopic dermatitis or whose skin has been prepped for surgery, appear to be at increased risk for these types of contact dermatoses.

Though there are other items in the differential, the configuration of the papulovesicular rash and the sole symptom of itching are essentially pathognomic for contact dermatitis. Besides the use of potent topical steroids for a few days, the real “cure” for this problem is for the patient to switch to “double-antibiotic” creams or ointments that do not include neomycin. 

ANSWER

The correct answer is an allergic reaction to a contactant, most likely the triple-antibiotic ointment (choice “d”).

Irritant reactions to tape adhesive (choice “a”) are extremely common. However, the resultant rash would have been confined to the linear areas where the tape touched his skin.

Dissolving sutures, such as those used in this case, can provoke a “suture granuloma”—essentially a foreign body reaction to the suture material (choice “b”). But this would have caused a focal area of swelling and redness, and very possibly a show of pus.

Postop wound infections (choice “c”) are also quite common. However, they would not manifest solely with itching in a papulovesicular rash surrounding the wound. Had infection developed, the redness would have been broad-based, with ill-defined margins, and the patient’s complaint would have been of pain, not itching. No vesicles would have been seen with bacterial infection.

DISCUSSION

This case illustrates the phenomenon of “treatment as problem,” in which the medication the patient applies becomes more problematic than the condition being addressed. Reactions to the neomycin in triple-antibiotic ointment are common but still provoke considerable worry on the part of patients and providers alike, especially when mistaken for “infection.”

This patient, like many, was dubious of the diagnosis, pointing out that he had used this same topical medication on many occasions without incident (though not recently). What he didn’t know is that it takes repeated exposure to a given allergen to develop T-memory cells that eventually begin to react. This same phenomenon is seen with poison ivy; patients will recall the ability, as a child, to practically wallow in poison ivy with impunity, making them doubtful about being allergic to it as an adult.

Neomycin, an aminoglycoside with a fairly wide spectrum of antibacterial activity, was first noted as a contact allergen in 1952. It is such a notorious offender that it was named Allergen of the Year in 2010 by the American Contact Dermatology Society.

For the past 20 years, 7% to 13% of patch tests surveyed were positive for neomycin. For reasons not entirely clear, Americans older than 60 are 150% more likely to experience a reaction to neomycin than are younger patients. (It could simply be that they’ve had more chances for exposure.)

In another interesting twist, the ointment vehicle appears to play a role. A reaction to this preparation is considerably more likely than to the same drug in other forms (eg, powders, solutions, creams). This is true of most medications, such as topical steroids, which are effectively self-occluded by this vehicle.

Persons with impaired barrier function, such as those with atopic dermatitis or whose skin has been prepped for surgery, appear to be at increased risk for these types of contact dermatoses.

Though there are other items in the differential, the configuration of the papulovesicular rash and the sole symptom of itching are essentially pathognomic for contact dermatitis. Besides the use of potent topical steroids for a few days, the real “cure” for this problem is for the patient to switch to “double-antibiotic” creams or ointments that do not include neomycin. 

Issue
Clinician Reviews - 24(10)
Issue
Clinician Reviews - 24(10)
Page Number
16-17
Page Number
16-17
Publications
Publications
Topics
Article Type
Display Headline
Postop Patient Reports “Wound Infection”
Display Headline
Postop Patient Reports “Wound Infection”
Legacy Keywords
sharply defined, bright pink, papulovesicular rash, triple-antibiotic ointment, neomycin, contact allergen
Legacy Keywords
sharply defined, bright pink, papulovesicular rash, triple-antibiotic ointment, neomycin, contact allergen
Sections
Questionnaire Body

 

 

A week ago, a 56-year-old man had a skin cancer surgically removed. Last night, he presented to an urgent care clinic for evaluation of a “wound infection” and received a prescription for double-strength trimethoprim/sulfa tablets (to be taken bid for 10 days). He is now in the dermatology office for follow-up. According to the patient, the problem manifested two days postop. There was no associated pain, only itching. The patient feels fine, with no fever or malaise, and there is no history of immunosuppression. He reports following his postop instructions well, changing his bandage daily and using triple-antibiotic ointment to dress the wound directly. The immediate peri-incisional area is indicated as the source of the problem. Surrounding the incision, which is healing well otherwise, is a sharply defined, bright pink, papulovesicular rash on a slightly edematous base. There is no tenderness on palpation, and no purulent material can be expressed from the wound. The area is only slightly warmer than the surrounding skin.

 

Disallow All Ads
Article PDF Media

Rapidly Enlarging Noduloulcerative Lesions

Article Type
Changed
Thu, 01/10/2019 - 13:17
Display Headline
Rapidly Enlarging Noduloulcerative Lesions

The Diagnosis: Lues Maligna

Biopsy revealed dense nodular aggregates of lymphocytes, histiocytes, and abundant plasma cells in both the superficial and deep dermis (Figure 1). There were perivascular and periadnexal aggregates of lymphocytes, histiocytes, and numerous plasma cells (Figure 2). Special stains for organisms, including Warthin-Starry silver, Giemsa, acid-fast bacilli, Gomori methenamine-silver, and Brown-Brenn stains were negative. Immunoperoxidase stain for Treponema pallidum also was negative. The patient’s rapid plasma reagin titer at the time of the fourth biopsy was 1:256, and appropriate treatment with penicillin resulted in complete clearance of the lesions in 3 to 4 weeks.

Figure 1. Superficial and deep perivascular and periadnexal lymphohistiocytic infiltrate (H&E, original magnification ×2).

Figure 2. Perivascular and periadnexal aggregates of lymphocytes, histiocytes, and numerous plasma cells (H&E, original magnification ×20).

Syphilis is caused by T pallidum. Three stages typically are identified in immunocompetent hosts: primary, secondary, and tertiary syphilis. Immunocompromised patients with human immunodeficiency virus (HIV) infection may have unusual presentations.

Lues maligna is used to describe a rare noduloulcerative form of secondary syphilis.1 It was first described in 18592 and has been associated with other disorders such as diabetes mellitus3 and chronic alcoholism.4 Patients usually are gravely ill and develop polymorphic ulcerating lesions. Facial and scalp involvement are common, but patients typically do not have palmoplantar involvement in conventional presentations of secondary syphilis.

A scanning view of a punch biopsy from our patient revealed irregular acanthosis of the epidermis with long and thin rete pegs, a bandlike infiltrate at the dermoepidermal junction, and a dense superficial and deep perivascular and periadnexal infiltrate. The histologic differential diagnosis includes pyoderma gangrenosum, vasculitis, lymphoma, leishmaniasis, leprosy, yaws, and mycobacterial or fungal infections.

The Centers for Disease Control and Prevention recommends screening of all HIV-positive indivi-duals for syphilis, and all sexually active individuals with syphilis should be screened for HIV.5 If clinical examination and findings suggest syphilis in the presence of negative serologic testing, then direct fluorescence assay for T pallidum staining of lesions, exudates or biopsy, or dark-field microscopic examination should be performed. In our case, dark-field microscopy was not performed and serologic tests were negative at presentation. Silver stains can detect T pallidum in tissue specimens, though detection may not be possible late in the course of disease.6

The morphology and rapid response to treatment confirmed the diagnosis in our patient. The incidence of syphilis in HIV-positive patients has risen substantially in the last 2 decades. This case illustrates an uncommon presentation that is increasing in prevalence.

References

1. Fisher DA, Chang LW, Tuffanelli DL. Lues maligna: presentation of a case and review of the literature. Arch Dermatol. 1969;99:70-73.

2. Passoni LF, de Menezes JA, Ribeiro SR, et al. Lues maligna in an HIV-infected patient [published online ahead of print March 30, 2005]. Rev Soc Bras Med Trop. 2005;38:181-184.

3. Hofmann UB, Hund M, Bröcker EB, et al. Lues maligna in a female patient with diabetes [in German]. J Dtsch Dermatol Ges. 2005;3:780-782.

4. Bayramgürler D, Bilen N, Yildiz K, et al. Lues maligna in a chronic alcoholic patient. J Dermatol. 2005;32:217-219.

5. Centers for Disease Control. Recommendations for diagnosing and treating syphilis in HIV-infected patients. MMWR Morb Mortal Wkly Rep. 1988;37:600-602.

6. Mannara GM. Bilateral secondary syphilis of the tonsil. J Laryngol Otol. 1999;113:1125-1127.

Article PDF
Author and Disclosure Information

Liaqat Ali, MD; Thomas Helm, MD; Brook Brouha, MD; Julie Gladsjo, MD; Clay Cockerell, MD

Dr. Ali is from the Department of Dermatology, Wayne State University, Dearborn, Michigan, and Pinkus Dermatopathology Laboratory, Monroe, Michigan. Dr. Helm is from the State University of New York at Buffalo. Dr. Brouha was from and Drs. Gladsjo and Cockerell are from the University of Texas Southwestern Medical Center, Dallas. Dr. Brouha currently is in private practice, La Jolla, California.

The authors report no conflict of interest.

Correspondence: Liaqat Ali, MD, 1314 N Macomb St, Monroe, MI 48162 (liaqali@gmail.com).

Issue
Cutis - 94(3)
Publications
Topics
Page Number
E20-E22
Legacy Keywords
syphilis, lues maligna, dermal nodular infiltrate
Sections
Author and Disclosure Information

Liaqat Ali, MD; Thomas Helm, MD; Brook Brouha, MD; Julie Gladsjo, MD; Clay Cockerell, MD

Dr. Ali is from the Department of Dermatology, Wayne State University, Dearborn, Michigan, and Pinkus Dermatopathology Laboratory, Monroe, Michigan. Dr. Helm is from the State University of New York at Buffalo. Dr. Brouha was from and Drs. Gladsjo and Cockerell are from the University of Texas Southwestern Medical Center, Dallas. Dr. Brouha currently is in private practice, La Jolla, California.

The authors report no conflict of interest.

Correspondence: Liaqat Ali, MD, 1314 N Macomb St, Monroe, MI 48162 (liaqali@gmail.com).

Author and Disclosure Information

Liaqat Ali, MD; Thomas Helm, MD; Brook Brouha, MD; Julie Gladsjo, MD; Clay Cockerell, MD

Dr. Ali is from the Department of Dermatology, Wayne State University, Dearborn, Michigan, and Pinkus Dermatopathology Laboratory, Monroe, Michigan. Dr. Helm is from the State University of New York at Buffalo. Dr. Brouha was from and Drs. Gladsjo and Cockerell are from the University of Texas Southwestern Medical Center, Dallas. Dr. Brouha currently is in private practice, La Jolla, California.

The authors report no conflict of interest.

Correspondence: Liaqat Ali, MD, 1314 N Macomb St, Monroe, MI 48162 (liaqali@gmail.com).

Article PDF
Article PDF

The Diagnosis: Lues Maligna

Biopsy revealed dense nodular aggregates of lymphocytes, histiocytes, and abundant plasma cells in both the superficial and deep dermis (Figure 1). There were perivascular and periadnexal aggregates of lymphocytes, histiocytes, and numerous plasma cells (Figure 2). Special stains for organisms, including Warthin-Starry silver, Giemsa, acid-fast bacilli, Gomori methenamine-silver, and Brown-Brenn stains were negative. Immunoperoxidase stain for Treponema pallidum also was negative. The patient’s rapid plasma reagin titer at the time of the fourth biopsy was 1:256, and appropriate treatment with penicillin resulted in complete clearance of the lesions in 3 to 4 weeks.

Figure 1. Superficial and deep perivascular and periadnexal lymphohistiocytic infiltrate (H&E, original magnification ×2).

Figure 2. Perivascular and periadnexal aggregates of lymphocytes, histiocytes, and numerous plasma cells (H&E, original magnification ×20).

Syphilis is caused by T pallidum. Three stages typically are identified in immunocompetent hosts: primary, secondary, and tertiary syphilis. Immunocompromised patients with human immunodeficiency virus (HIV) infection may have unusual presentations.

Lues maligna is used to describe a rare noduloulcerative form of secondary syphilis.1 It was first described in 18592 and has been associated with other disorders such as diabetes mellitus3 and chronic alcoholism.4 Patients usually are gravely ill and develop polymorphic ulcerating lesions. Facial and scalp involvement are common, but patients typically do not have palmoplantar involvement in conventional presentations of secondary syphilis.

A scanning view of a punch biopsy from our patient revealed irregular acanthosis of the epidermis with long and thin rete pegs, a bandlike infiltrate at the dermoepidermal junction, and a dense superficial and deep perivascular and periadnexal infiltrate. The histologic differential diagnosis includes pyoderma gangrenosum, vasculitis, lymphoma, leishmaniasis, leprosy, yaws, and mycobacterial or fungal infections.

The Centers for Disease Control and Prevention recommends screening of all HIV-positive indivi-duals for syphilis, and all sexually active individuals with syphilis should be screened for HIV.5 If clinical examination and findings suggest syphilis in the presence of negative serologic testing, then direct fluorescence assay for T pallidum staining of lesions, exudates or biopsy, or dark-field microscopic examination should be performed. In our case, dark-field microscopy was not performed and serologic tests were negative at presentation. Silver stains can detect T pallidum in tissue specimens, though detection may not be possible late in the course of disease.6

The morphology and rapid response to treatment confirmed the diagnosis in our patient. The incidence of syphilis in HIV-positive patients has risen substantially in the last 2 decades. This case illustrates an uncommon presentation that is increasing in prevalence.

The Diagnosis: Lues Maligna

Biopsy revealed dense nodular aggregates of lymphocytes, histiocytes, and abundant plasma cells in both the superficial and deep dermis (Figure 1). There were perivascular and periadnexal aggregates of lymphocytes, histiocytes, and numerous plasma cells (Figure 2). Special stains for organisms, including Warthin-Starry silver, Giemsa, acid-fast bacilli, Gomori methenamine-silver, and Brown-Brenn stains were negative. Immunoperoxidase stain for Treponema pallidum also was negative. The patient’s rapid plasma reagin titer at the time of the fourth biopsy was 1:256, and appropriate treatment with penicillin resulted in complete clearance of the lesions in 3 to 4 weeks.

Figure 1. Superficial and deep perivascular and periadnexal lymphohistiocytic infiltrate (H&E, original magnification ×2).

Figure 2. Perivascular and periadnexal aggregates of lymphocytes, histiocytes, and numerous plasma cells (H&E, original magnification ×20).

Syphilis is caused by T pallidum. Three stages typically are identified in immunocompetent hosts: primary, secondary, and tertiary syphilis. Immunocompromised patients with human immunodeficiency virus (HIV) infection may have unusual presentations.

Lues maligna is used to describe a rare noduloulcerative form of secondary syphilis.1 It was first described in 18592 and has been associated with other disorders such as diabetes mellitus3 and chronic alcoholism.4 Patients usually are gravely ill and develop polymorphic ulcerating lesions. Facial and scalp involvement are common, but patients typically do not have palmoplantar involvement in conventional presentations of secondary syphilis.

A scanning view of a punch biopsy from our patient revealed irregular acanthosis of the epidermis with long and thin rete pegs, a bandlike infiltrate at the dermoepidermal junction, and a dense superficial and deep perivascular and periadnexal infiltrate. The histologic differential diagnosis includes pyoderma gangrenosum, vasculitis, lymphoma, leishmaniasis, leprosy, yaws, and mycobacterial or fungal infections.

The Centers for Disease Control and Prevention recommends screening of all HIV-positive indivi-duals for syphilis, and all sexually active individuals with syphilis should be screened for HIV.5 If clinical examination and findings suggest syphilis in the presence of negative serologic testing, then direct fluorescence assay for T pallidum staining of lesions, exudates or biopsy, or dark-field microscopic examination should be performed. In our case, dark-field microscopy was not performed and serologic tests were negative at presentation. Silver stains can detect T pallidum in tissue specimens, though detection may not be possible late in the course of disease.6

The morphology and rapid response to treatment confirmed the diagnosis in our patient. The incidence of syphilis in HIV-positive patients has risen substantially in the last 2 decades. This case illustrates an uncommon presentation that is increasing in prevalence.

References

1. Fisher DA, Chang LW, Tuffanelli DL. Lues maligna: presentation of a case and review of the literature. Arch Dermatol. 1969;99:70-73.

2. Passoni LF, de Menezes JA, Ribeiro SR, et al. Lues maligna in an HIV-infected patient [published online ahead of print March 30, 2005]. Rev Soc Bras Med Trop. 2005;38:181-184.

3. Hofmann UB, Hund M, Bröcker EB, et al. Lues maligna in a female patient with diabetes [in German]. J Dtsch Dermatol Ges. 2005;3:780-782.

4. Bayramgürler D, Bilen N, Yildiz K, et al. Lues maligna in a chronic alcoholic patient. J Dermatol. 2005;32:217-219.

5. Centers for Disease Control. Recommendations for diagnosing and treating syphilis in HIV-infected patients. MMWR Morb Mortal Wkly Rep. 1988;37:600-602.

6. Mannara GM. Bilateral secondary syphilis of the tonsil. J Laryngol Otol. 1999;113:1125-1127.

References

1. Fisher DA, Chang LW, Tuffanelli DL. Lues maligna: presentation of a case and review of the literature. Arch Dermatol. 1969;99:70-73.

2. Passoni LF, de Menezes JA, Ribeiro SR, et al. Lues maligna in an HIV-infected patient [published online ahead of print March 30, 2005]. Rev Soc Bras Med Trop. 2005;38:181-184.

3. Hofmann UB, Hund M, Bröcker EB, et al. Lues maligna in a female patient with diabetes [in German]. J Dtsch Dermatol Ges. 2005;3:780-782.

4. Bayramgürler D, Bilen N, Yildiz K, et al. Lues maligna in a chronic alcoholic patient. J Dermatol. 2005;32:217-219.

5. Centers for Disease Control. Recommendations for diagnosing and treating syphilis in HIV-infected patients. MMWR Morb Mortal Wkly Rep. 1988;37:600-602.

6. Mannara GM. Bilateral secondary syphilis of the tonsil. J Laryngol Otol. 1999;113:1125-1127.

Issue
Cutis - 94(3)
Issue
Cutis - 94(3)
Page Number
E20-E22
Page Number
E20-E22
Publications
Publications
Topics
Article Type
Display Headline
Rapidly Enlarging Noduloulcerative Lesions
Display Headline
Rapidly Enlarging Noduloulcerative Lesions
Legacy Keywords
syphilis, lues maligna, dermal nodular infiltrate
Legacy Keywords
syphilis, lues maligna, dermal nodular infiltrate
Sections
Questionnaire Body

A 43-year-old man presented with a rapidly enlarging ulcerated nodule on the right ankle with a necrotic and crusted center. He also had multiple red-brown papules on the trunk and extremities. Some of these lesions had central erosions, while others had surface scale. He was known to be human immunodeficiency virus positive but had no lymphadenopathy. The CD4+ lymphocyte count was 153 cells/mm3 (reference range, 400–1600 cells/mm3) and he was on highly active antiretroviral therapy.
Article Source

PURLs Copyright

Inside the Article

Article PDF Media

Man Awakens With "Fluttering" in His Chest

Article Type
Changed
Tue, 12/13/2016 - 12:08
Display Headline
Man Awakens With "Fluttering" in His Chest

ANSWER

This ECG is consistent with coarse atrial fibrillation with a rapid ventricular response and a nonspecific T-wave abnormality. The patient’s presentation is strongly suggestive of lone atrial fibrillation: This was the first incidence, it occurred in the absence of an existing heart condition, and it presented with an abrupt onset of increased heart rate and dyspnea.

Lone atrial fibrillation most commonly occurs in men in their 40s and 50s. It is vagally mediated, occurring during sleep or relaxation and after food and/or alcohol consumption.

The patient was cardioverted to normal sinus rhythm in the ED without difficulty, and follow-up was arranged. 

References

Article PDF
Author and Disclosure Information

Lyle W. Larson, PhD, PA-C, is clinical faculty in the Department of Medicine, Division of Cardiology, Cardiac Electrophysiology, at the University of Washington, Seattle.

Issue
Clinician Reviews - 24(10)
Publications
Topics
Page Number
18,21
Legacy Keywords
Fluttering,shortness of breath,rapid heart rate,coarse atrial fibrillation, rapid ventricular response, nonspecific T-wave abnormality
Sections
Author and Disclosure Information

Lyle W. Larson, PhD, PA-C, is clinical faculty in the Department of Medicine, Division of Cardiology, Cardiac Electrophysiology, at the University of Washington, Seattle.

Author and Disclosure Information

Lyle W. Larson, PhD, PA-C, is clinical faculty in the Department of Medicine, Division of Cardiology, Cardiac Electrophysiology, at the University of Washington, Seattle.

Article PDF
Article PDF

ANSWER

This ECG is consistent with coarse atrial fibrillation with a rapid ventricular response and a nonspecific T-wave abnormality. The patient’s presentation is strongly suggestive of lone atrial fibrillation: This was the first incidence, it occurred in the absence of an existing heart condition, and it presented with an abrupt onset of increased heart rate and dyspnea.

Lone atrial fibrillation most commonly occurs in men in their 40s and 50s. It is vagally mediated, occurring during sleep or relaxation and after food and/or alcohol consumption.

The patient was cardioverted to normal sinus rhythm in the ED without difficulty, and follow-up was arranged. 

ANSWER

This ECG is consistent with coarse atrial fibrillation with a rapid ventricular response and a nonspecific T-wave abnormality. The patient’s presentation is strongly suggestive of lone atrial fibrillation: This was the first incidence, it occurred in the absence of an existing heart condition, and it presented with an abrupt onset of increased heart rate and dyspnea.

Lone atrial fibrillation most commonly occurs in men in their 40s and 50s. It is vagally mediated, occurring during sleep or relaxation and after food and/or alcohol consumption.

The patient was cardioverted to normal sinus rhythm in the ED without difficulty, and follow-up was arranged. 

References

References

Issue
Clinician Reviews - 24(10)
Issue
Clinician Reviews - 24(10)
Page Number
18,21
Page Number
18,21
Publications
Publications
Topics
Article Type
Display Headline
Man Awakens With "Fluttering" in His Chest
Display Headline
Man Awakens With "Fluttering" in His Chest
Legacy Keywords
Fluttering,shortness of breath,rapid heart rate,coarse atrial fibrillation, rapid ventricular response, nonspecific T-wave abnormality
Legacy Keywords
Fluttering,shortness of breath,rapid heart rate,coarse atrial fibrillation, rapid ventricular response, nonspecific T-wave abnormality
Sections
Questionnaire Body

A 56-year-old man presents to the emergency department (ED) complaining of shortness of breath and a rapid heart rate. He went to bed at his regular time (10:30 pm) last night and woke up at 3:30 am with a fluttering sensation in his chest. He checked his pulse; it was 120 beats/min. Alarmed, he got out of bed and noted he was short of breath as he walked to the bathroom. He went back to bed, but after approximately 20 minutes without relief, he decided to call his son to take him to the ED. The time from onset of symptoms until arrival at the ED was two hours. During that time, his symptoms did not change. When you examine the patient, he states that he is typically in excellent health and has never experienced either shortness of breath or a rapid heart rate before. He denies a history of cardiac or pulmonary disease and has never had chest pain, syncope, or near-syncope. Medical history is unremarkable. Surgical history is remarkable for a tonsillectomy in childhood and an appendectomy for acute appendicitis at age 18. The patient has no known drug allergies and is taking ibuprofen for a recent ankle sprain but is on no other medications. He works as a certified public accountant and has a sedentary lifestyle. He drinks two to three glasses of wine each evening, does not smoke, and denies recreational or naturopathic medication use. He is a widower (his wife died of breast cancer at age 44) and has one son who lives in the same housing complex. The review of systems is remarkable for a recent left ankle sprain, which occurred when the patient slipped on the carpet at home. Vital signs include a blood pressure of 144/84 mm Hg; pulse, 130 beats/min; respiratory rate, 18 breaths/min-1; O2 saturation, 98%; and temperature, 98.9°F. His height is 5 ft 9 in and his weight, 223 lb. The physical exam reveals an obese white male in mild distress. The HEENT exam reveals corrective lenses and the absence of tonsils. The neck shows no evidence of thyromegaly or jugular venous distention. The lungs are clear in all fields. The cardiac rhythm is irregular with a rate of 130 beats/min. There are no murmurs or extra heart sounds audible. The abdomen is obese and nontender, with no palpable masses. An old surgical scar is evident in the right lower quadrant, consistent with his history of an appendectomy. The lower extremities show no evidence of peripheral edema. Mild discomfort is present with examination of the left ankle. Peripheral pulses are strong and equal, and the neurologic exam is intact. An ECG is obtained that reveals a ventricular rate of 131 beats/min; PR interval, not measured; QRS duration, 82 ms; QT/QTc interval, 374/552 ms; no P axis; R axis, 68°; and T axis, 36°. What is your interpretation of this ECG?
Article Source

PURLs Copyright

Inside the Article

Article PDF Media

Yellowish Papulonodular Periorbital Eruption

Article Type
Changed
Thu, 01/10/2019 - 13:17
Display Headline
Yellowish Papulonodular Periorbital Eruption

The Diagnosis: Adult-Onset Xanthogranuloma

Biopsies of the lesions on the neck (Figure 1) and back were performed. Histologic analyses revealed a diffuse dermatitis consisting of foamy histiocytes admixed with a few Touton-type giant cells in the dermis (Figure 2), which was associated with an inflammatory infiltrate of eosinophils and lymphocytes. Laboratory investigations revealed mild thrombocytopenia with a platelet count of 134×109/L (reference range, 140–440×109/L). Other investigations including biochemistry, lipid, serum electrophoresis, and chest radiogram were normal. A bone marrow trephine biopsy and flow cytometry were performed and were normal. Magnetic resonance imaging revealed periorbital soft-tissue masses that did not extend into the orbits.

Figure 1. Firm plaques with a yellowish tinge over the side of the neck

Figure 2. A dense infiltrate of foamy histiocytes in the dermis (A) associated with a Touton-type giant cell (arrow) and an inflammatory infiltrate consisting of eosinophils and lymphocytes (B)(both H&E, original magnifications ×200 and ×400).

Adult-onset xanthogranuloma (AXG) is a rare disease entity, usually presenting in the third to fourth decades of life. The condition typically presents as a red to yellow-brown nodular cutaneous lesion located on the scalp, face, neck, trunk, or limbs. The presentation typically consists of a solitary lesion, occurring in 70% to 89% of cases,1 but more rarely, as in this case, lesions can be multiple or even disseminated.

Histologically, AXG presents as a dense, well-circumscribed, histiocytic infiltrate consisting of lipophages possessing foamy cytoplasm and giant cells. The presence of histiocytic giant cells differentiates AXG from xanthelasma, a clinical differential diagnosis in this case, and xanthoma. In AXG, there are 4 main types of histiocytes: xanthomatized, spindle shaped, vacuolated, and oncocytic.2 They can be seen in variable proportions, together with different types of giant cells (eg, Touton, foreign body, ground glass, nonspecific). A mixed infiltrate of eosinophils, lymphocytes, plasma cells, and neutrophils also may be seen scattered throughout the lesion.2

Correlating with the clinical and histological features of xanthogranuloma, the firm plaques and nodules represent the dense dermal infiltration of histiocytes that may extend into the subcutis. The lesions demonstrate a time-dependent progression both clinically and histologically. Early lesions are comprised of a dense monomorphous nonlipid histiocytic inflammatory infiltrate, and they clinically appear more erythematous. In mature lesions, as in our patient, the infiltrate is predominantly composed of lipid-laden histiocytes with associated Touton giant cells. They appear more yellowish on clinical presentation.

Adult-onset xanthogranuloma is part of a rare heterogenous group of disorders termed adult orbital xanthogranulomatous disease, which includes 3 other syndromes: necrobiotic xanthogranuloma, adult-onset asthma and periocular xanthogranuloma, and Erdheim-Chester disease.

Necrobiotic xanthogranuloma is clinically characterized by the presence of subcutaneous lesions that ulcerate in approximately 40% to 50% of cases and is histologically characterized by necrobiosis with palisading epithelioid histiocytes. It also is systemically associated with paraproteinemia and multiple myeloma.3

Adult-onset asthma and periocular xanthogranuloma is characterized by yellowish papules and nodules predominantly over the lower eyelids that are histologically comprised of lymphoid follicles with reactive germinal centers. It is associated with asthma, which normally is severe and often occurs almost simultaneously with the periorbital lesions.4

There are no definite diagnostic criteria for Erdheim-Chester disease and the diagnosis is usually based on radiologic findings of osteosclerosis and histopathologic evidence of foamy histiocytic infiltration. Systemic manifestations are common with lymphohistiocytic infiltration of the heart, lungs, pericardium, bones, and intestines. Prognosis is uniformly dismal.

Based on the clinical presentation of a nonulcerative papulonodular eruption and the absence of systemic involvement including asthma, we made the diagnosis of AXG. In view of the heterogeneity among these clinical entities as well as the time-based evolution of the lesions involved, we continued to monitor the patient for 2 years and there was no development of other systemic manifestations and hematologic abnormalities.

In contrast to the more common form of juvenile-onset xanthogranuloma, the adult type is not associated with widespread visceral lesions. Hence extensive screening investigations for systemic disease generally are not necessary. Another difference is that AXG has been associated with hematologic abnormalities, including essential thrombocytosis, chronic lymphocytic leukemia, large B-cell lymphoma, and monoclonal gammopathy.5,6 In our patient, the presence of thrombocytopenia and older age caused us to be concerned about an associated hematologic malignancy; therefore, a bone marrow biopsy and flow cytometry were performed.

Adult-onset xanthogranuloma typically is an asymptomatic and self-healing disease and therefore treatment usually is not required. Spontaneous regression of xanthogranuloma was observed to occur in 54% of cases with a median duration of 22 months,7 though lesions were noted to last as long as 15 years.8 When treatment is necessary, a combination of local and systemic corticosteroids, cytotoxic agents, and radiotherapy have been routinely used. In particular, intralesional corticosteroid therapy has been found to be efficacious in controlling symptoms and signs of AXG affecting the eyelids and orbits while avoiding the systemic side effects of other agents.9

 

 

Because our patient’s lesions were longstanding and disfiguring, we opted for active intervention with intralesional triamcinolone, which resulted in only a slight reduction in size of the lesions. The lesions remain largely unchanged in 2 years of follow-up.

References

1. Chang SE, Cho S, Choi JC, et al. Clinicohistopathologic comparison of adult type and juvenile type xanthogranulomas in Korea. J Dermatol. 2001;28:413-418.

2. Gelmetti C. Non-Langerhans cell histiocytosis. In: Goldsmith LA, Katz SI, Gilchrest BA, et al, eds. Fitzpatrick’s Dermatology in General Medicine. 8th ed. New York, NY: McGraw-Hill; 2012.

3. Mehregan DA, Winkelmann RK. Necrobiotic xanthogranuloma. Arch Dermatol. 1992;128:94-100.

4. Jakobiec FA, Mills MD, Hidayat AA, et al. Periocular xanthogranulomas associated with severe adult-onset asthma. Trans Am Ophthalmol Soc. 1993;91:99-125.

5. Shoo BA, Shinkai K, McCalmont TH, et al. Xanthogranulomas associated with hematologic malignancy in adulthood. J Am Acad Dermatol. 2008;59:488-493.

6. Chiou CC, Wang PN, Yang LC, et al. Disseminated xanthogranulomas associated with adult T-cell leukaemia/lymphoma: a case report and review the association of haematologic malignancies. J Eur Acad Dermatol Venereol. 2007;21:532-535.

7. Robinson HM, Harmon CE, Firminger HI. Multiple lipoidal histiocytomas with regression. Arch Dermatol. 1963;88:660-667.

8. Winkelmann RK. Cutaneous syndromes of non-X histiocytosis: a review of the macrophage-histiocyte diseases of the skin. Arch Dermatol. 1981;117:667-672.

9. Elner VM, Mintz R, Demirci H, et al. Local corticosteroid treatment of eyelid and orbital xanthogranuloma. Ophthal Plast Reconstr Surg. 2006;22:36-40.

Article PDF
Author and Disclosure Information

Brian Chia, MBBS, MRCP; Hong Liang Tey, MBBS, MRCP, MRCPS

Dr. Chia is from the Department of General Medicine, Tan Tock Seng Hospital, Singapore. Dr. Tey is from the National Skin Centre, Singapore.

The authors report no conflict of interest.

Correspondence: Hong Liang Tey, MBBS, MRCP, MRCPS, 1 Mandalay Rd, Singapore 308205 (teyhongliang111@yahoo.com).

Issue
Cutis - 94(3)
Publications
Topics
Page Number
E7-E10
Legacy Keywords
xanthogranuloma, adult, histiocytosis, periorbital
Sections
Author and Disclosure Information

Brian Chia, MBBS, MRCP; Hong Liang Tey, MBBS, MRCP, MRCPS

Dr. Chia is from the Department of General Medicine, Tan Tock Seng Hospital, Singapore. Dr. Tey is from the National Skin Centre, Singapore.

The authors report no conflict of interest.

Correspondence: Hong Liang Tey, MBBS, MRCP, MRCPS, 1 Mandalay Rd, Singapore 308205 (teyhongliang111@yahoo.com).

Author and Disclosure Information

Brian Chia, MBBS, MRCP; Hong Liang Tey, MBBS, MRCP, MRCPS

Dr. Chia is from the Department of General Medicine, Tan Tock Seng Hospital, Singapore. Dr. Tey is from the National Skin Centre, Singapore.

The authors report no conflict of interest.

Correspondence: Hong Liang Tey, MBBS, MRCP, MRCPS, 1 Mandalay Rd, Singapore 308205 (teyhongliang111@yahoo.com).

Article PDF
Article PDF

The Diagnosis: Adult-Onset Xanthogranuloma

Biopsies of the lesions on the neck (Figure 1) and back were performed. Histologic analyses revealed a diffuse dermatitis consisting of foamy histiocytes admixed with a few Touton-type giant cells in the dermis (Figure 2), which was associated with an inflammatory infiltrate of eosinophils and lymphocytes. Laboratory investigations revealed mild thrombocytopenia with a platelet count of 134×109/L (reference range, 140–440×109/L). Other investigations including biochemistry, lipid, serum electrophoresis, and chest radiogram were normal. A bone marrow trephine biopsy and flow cytometry were performed and were normal. Magnetic resonance imaging revealed periorbital soft-tissue masses that did not extend into the orbits.

Figure 1. Firm plaques with a yellowish tinge over the side of the neck

Figure 2. A dense infiltrate of foamy histiocytes in the dermis (A) associated with a Touton-type giant cell (arrow) and an inflammatory infiltrate consisting of eosinophils and lymphocytes (B)(both H&E, original magnifications ×200 and ×400).

Adult-onset xanthogranuloma (AXG) is a rare disease entity, usually presenting in the third to fourth decades of life. The condition typically presents as a red to yellow-brown nodular cutaneous lesion located on the scalp, face, neck, trunk, or limbs. The presentation typically consists of a solitary lesion, occurring in 70% to 89% of cases,1 but more rarely, as in this case, lesions can be multiple or even disseminated.

Histologically, AXG presents as a dense, well-circumscribed, histiocytic infiltrate consisting of lipophages possessing foamy cytoplasm and giant cells. The presence of histiocytic giant cells differentiates AXG from xanthelasma, a clinical differential diagnosis in this case, and xanthoma. In AXG, there are 4 main types of histiocytes: xanthomatized, spindle shaped, vacuolated, and oncocytic.2 They can be seen in variable proportions, together with different types of giant cells (eg, Touton, foreign body, ground glass, nonspecific). A mixed infiltrate of eosinophils, lymphocytes, plasma cells, and neutrophils also may be seen scattered throughout the lesion.2

Correlating with the clinical and histological features of xanthogranuloma, the firm plaques and nodules represent the dense dermal infiltration of histiocytes that may extend into the subcutis. The lesions demonstrate a time-dependent progression both clinically and histologically. Early lesions are comprised of a dense monomorphous nonlipid histiocytic inflammatory infiltrate, and they clinically appear more erythematous. In mature lesions, as in our patient, the infiltrate is predominantly composed of lipid-laden histiocytes with associated Touton giant cells. They appear more yellowish on clinical presentation.

Adult-onset xanthogranuloma is part of a rare heterogenous group of disorders termed adult orbital xanthogranulomatous disease, which includes 3 other syndromes: necrobiotic xanthogranuloma, adult-onset asthma and periocular xanthogranuloma, and Erdheim-Chester disease.

Necrobiotic xanthogranuloma is clinically characterized by the presence of subcutaneous lesions that ulcerate in approximately 40% to 50% of cases and is histologically characterized by necrobiosis with palisading epithelioid histiocytes. It also is systemically associated with paraproteinemia and multiple myeloma.3

Adult-onset asthma and periocular xanthogranuloma is characterized by yellowish papules and nodules predominantly over the lower eyelids that are histologically comprised of lymphoid follicles with reactive germinal centers. It is associated with asthma, which normally is severe and often occurs almost simultaneously with the periorbital lesions.4

There are no definite diagnostic criteria for Erdheim-Chester disease and the diagnosis is usually based on radiologic findings of osteosclerosis and histopathologic evidence of foamy histiocytic infiltration. Systemic manifestations are common with lymphohistiocytic infiltration of the heart, lungs, pericardium, bones, and intestines. Prognosis is uniformly dismal.

Based on the clinical presentation of a nonulcerative papulonodular eruption and the absence of systemic involvement including asthma, we made the diagnosis of AXG. In view of the heterogeneity among these clinical entities as well as the time-based evolution of the lesions involved, we continued to monitor the patient for 2 years and there was no development of other systemic manifestations and hematologic abnormalities.

In contrast to the more common form of juvenile-onset xanthogranuloma, the adult type is not associated with widespread visceral lesions. Hence extensive screening investigations for systemic disease generally are not necessary. Another difference is that AXG has been associated with hematologic abnormalities, including essential thrombocytosis, chronic lymphocytic leukemia, large B-cell lymphoma, and monoclonal gammopathy.5,6 In our patient, the presence of thrombocytopenia and older age caused us to be concerned about an associated hematologic malignancy; therefore, a bone marrow biopsy and flow cytometry were performed.

Adult-onset xanthogranuloma typically is an asymptomatic and self-healing disease and therefore treatment usually is not required. Spontaneous regression of xanthogranuloma was observed to occur in 54% of cases with a median duration of 22 months,7 though lesions were noted to last as long as 15 years.8 When treatment is necessary, a combination of local and systemic corticosteroids, cytotoxic agents, and radiotherapy have been routinely used. In particular, intralesional corticosteroid therapy has been found to be efficacious in controlling symptoms and signs of AXG affecting the eyelids and orbits while avoiding the systemic side effects of other agents.9

 

 

Because our patient’s lesions were longstanding and disfiguring, we opted for active intervention with intralesional triamcinolone, which resulted in only a slight reduction in size of the lesions. The lesions remain largely unchanged in 2 years of follow-up.

The Diagnosis: Adult-Onset Xanthogranuloma

Biopsies of the lesions on the neck (Figure 1) and back were performed. Histologic analyses revealed a diffuse dermatitis consisting of foamy histiocytes admixed with a few Touton-type giant cells in the dermis (Figure 2), which was associated with an inflammatory infiltrate of eosinophils and lymphocytes. Laboratory investigations revealed mild thrombocytopenia with a platelet count of 134×109/L (reference range, 140–440×109/L). Other investigations including biochemistry, lipid, serum electrophoresis, and chest radiogram were normal. A bone marrow trephine biopsy and flow cytometry were performed and were normal. Magnetic resonance imaging revealed periorbital soft-tissue masses that did not extend into the orbits.

Figure 1. Firm plaques with a yellowish tinge over the side of the neck

Figure 2. A dense infiltrate of foamy histiocytes in the dermis (A) associated with a Touton-type giant cell (arrow) and an inflammatory infiltrate consisting of eosinophils and lymphocytes (B)(both H&E, original magnifications ×200 and ×400).

Adult-onset xanthogranuloma (AXG) is a rare disease entity, usually presenting in the third to fourth decades of life. The condition typically presents as a red to yellow-brown nodular cutaneous lesion located on the scalp, face, neck, trunk, or limbs. The presentation typically consists of a solitary lesion, occurring in 70% to 89% of cases,1 but more rarely, as in this case, lesions can be multiple or even disseminated.

Histologically, AXG presents as a dense, well-circumscribed, histiocytic infiltrate consisting of lipophages possessing foamy cytoplasm and giant cells. The presence of histiocytic giant cells differentiates AXG from xanthelasma, a clinical differential diagnosis in this case, and xanthoma. In AXG, there are 4 main types of histiocytes: xanthomatized, spindle shaped, vacuolated, and oncocytic.2 They can be seen in variable proportions, together with different types of giant cells (eg, Touton, foreign body, ground glass, nonspecific). A mixed infiltrate of eosinophils, lymphocytes, plasma cells, and neutrophils also may be seen scattered throughout the lesion.2

Correlating with the clinical and histological features of xanthogranuloma, the firm plaques and nodules represent the dense dermal infiltration of histiocytes that may extend into the subcutis. The lesions demonstrate a time-dependent progression both clinically and histologically. Early lesions are comprised of a dense monomorphous nonlipid histiocytic inflammatory infiltrate, and they clinically appear more erythematous. In mature lesions, as in our patient, the infiltrate is predominantly composed of lipid-laden histiocytes with associated Touton giant cells. They appear more yellowish on clinical presentation.

Adult-onset xanthogranuloma is part of a rare heterogenous group of disorders termed adult orbital xanthogranulomatous disease, which includes 3 other syndromes: necrobiotic xanthogranuloma, adult-onset asthma and periocular xanthogranuloma, and Erdheim-Chester disease.

Necrobiotic xanthogranuloma is clinically characterized by the presence of subcutaneous lesions that ulcerate in approximately 40% to 50% of cases and is histologically characterized by necrobiosis with palisading epithelioid histiocytes. It also is systemically associated with paraproteinemia and multiple myeloma.3

Adult-onset asthma and periocular xanthogranuloma is characterized by yellowish papules and nodules predominantly over the lower eyelids that are histologically comprised of lymphoid follicles with reactive germinal centers. It is associated with asthma, which normally is severe and often occurs almost simultaneously with the periorbital lesions.4

There are no definite diagnostic criteria for Erdheim-Chester disease and the diagnosis is usually based on radiologic findings of osteosclerosis and histopathologic evidence of foamy histiocytic infiltration. Systemic manifestations are common with lymphohistiocytic infiltration of the heart, lungs, pericardium, bones, and intestines. Prognosis is uniformly dismal.

Based on the clinical presentation of a nonulcerative papulonodular eruption and the absence of systemic involvement including asthma, we made the diagnosis of AXG. In view of the heterogeneity among these clinical entities as well as the time-based evolution of the lesions involved, we continued to monitor the patient for 2 years and there was no development of other systemic manifestations and hematologic abnormalities.

In contrast to the more common form of juvenile-onset xanthogranuloma, the adult type is not associated with widespread visceral lesions. Hence extensive screening investigations for systemic disease generally are not necessary. Another difference is that AXG has been associated with hematologic abnormalities, including essential thrombocytosis, chronic lymphocytic leukemia, large B-cell lymphoma, and monoclonal gammopathy.5,6 In our patient, the presence of thrombocytopenia and older age caused us to be concerned about an associated hematologic malignancy; therefore, a bone marrow biopsy and flow cytometry were performed.

Adult-onset xanthogranuloma typically is an asymptomatic and self-healing disease and therefore treatment usually is not required. Spontaneous regression of xanthogranuloma was observed to occur in 54% of cases with a median duration of 22 months,7 though lesions were noted to last as long as 15 years.8 When treatment is necessary, a combination of local and systemic corticosteroids, cytotoxic agents, and radiotherapy have been routinely used. In particular, intralesional corticosteroid therapy has been found to be efficacious in controlling symptoms and signs of AXG affecting the eyelids and orbits while avoiding the systemic side effects of other agents.9

 

 

Because our patient’s lesions were longstanding and disfiguring, we opted for active intervention with intralesional triamcinolone, which resulted in only a slight reduction in size of the lesions. The lesions remain largely unchanged in 2 years of follow-up.

References

1. Chang SE, Cho S, Choi JC, et al. Clinicohistopathologic comparison of adult type and juvenile type xanthogranulomas in Korea. J Dermatol. 2001;28:413-418.

2. Gelmetti C. Non-Langerhans cell histiocytosis. In: Goldsmith LA, Katz SI, Gilchrest BA, et al, eds. Fitzpatrick’s Dermatology in General Medicine. 8th ed. New York, NY: McGraw-Hill; 2012.

3. Mehregan DA, Winkelmann RK. Necrobiotic xanthogranuloma. Arch Dermatol. 1992;128:94-100.

4. Jakobiec FA, Mills MD, Hidayat AA, et al. Periocular xanthogranulomas associated with severe adult-onset asthma. Trans Am Ophthalmol Soc. 1993;91:99-125.

5. Shoo BA, Shinkai K, McCalmont TH, et al. Xanthogranulomas associated with hematologic malignancy in adulthood. J Am Acad Dermatol. 2008;59:488-493.

6. Chiou CC, Wang PN, Yang LC, et al. Disseminated xanthogranulomas associated with adult T-cell leukaemia/lymphoma: a case report and review the association of haematologic malignancies. J Eur Acad Dermatol Venereol. 2007;21:532-535.

7. Robinson HM, Harmon CE, Firminger HI. Multiple lipoidal histiocytomas with regression. Arch Dermatol. 1963;88:660-667.

8. Winkelmann RK. Cutaneous syndromes of non-X histiocytosis: a review of the macrophage-histiocyte diseases of the skin. Arch Dermatol. 1981;117:667-672.

9. Elner VM, Mintz R, Demirci H, et al. Local corticosteroid treatment of eyelid and orbital xanthogranuloma. Ophthal Plast Reconstr Surg. 2006;22:36-40.

References

1. Chang SE, Cho S, Choi JC, et al. Clinicohistopathologic comparison of adult type and juvenile type xanthogranulomas in Korea. J Dermatol. 2001;28:413-418.

2. Gelmetti C. Non-Langerhans cell histiocytosis. In: Goldsmith LA, Katz SI, Gilchrest BA, et al, eds. Fitzpatrick’s Dermatology in General Medicine. 8th ed. New York, NY: McGraw-Hill; 2012.

3. Mehregan DA, Winkelmann RK. Necrobiotic xanthogranuloma. Arch Dermatol. 1992;128:94-100.

4. Jakobiec FA, Mills MD, Hidayat AA, et al. Periocular xanthogranulomas associated with severe adult-onset asthma. Trans Am Ophthalmol Soc. 1993;91:99-125.

5. Shoo BA, Shinkai K, McCalmont TH, et al. Xanthogranulomas associated with hematologic malignancy in adulthood. J Am Acad Dermatol. 2008;59:488-493.

6. Chiou CC, Wang PN, Yang LC, et al. Disseminated xanthogranulomas associated with adult T-cell leukaemia/lymphoma: a case report and review the association of haematologic malignancies. J Eur Acad Dermatol Venereol. 2007;21:532-535.

7. Robinson HM, Harmon CE, Firminger HI. Multiple lipoidal histiocytomas with regression. Arch Dermatol. 1963;88:660-667.

8. Winkelmann RK. Cutaneous syndromes of non-X histiocytosis: a review of the macrophage-histiocyte diseases of the skin. Arch Dermatol. 1981;117:667-672.

9. Elner VM, Mintz R, Demirci H, et al. Local corticosteroid treatment of eyelid and orbital xanthogranuloma. Ophthal Plast Reconstr Surg. 2006;22:36-40.

Issue
Cutis - 94(3)
Issue
Cutis - 94(3)
Page Number
E7-E10
Page Number
E7-E10
Publications
Publications
Topics
Article Type
Display Headline
Yellowish Papulonodular Periorbital Eruption
Display Headline
Yellowish Papulonodular Periorbital Eruption
Legacy Keywords
xanthogranuloma, adult, histiocytosis, periorbital
Legacy Keywords
xanthogranuloma, adult, histiocytosis, periorbital
Sections
Questionnaire Body

A 66-year-old woman with a history of type 2 diabetes mellitus and mild dyslipidemia presented with persistent lesions over the eyelids and cheeks of 10 years’ duration. Systemic review was unremarkable. There was no family or personal history of atopy, asthma, or other dermatologic disorders. Physical examination revealed confluent yellowish plaques and nodules over the periorbital regions as well as yellowish plaques over the neck and back. The lesions were firm to palpation and the epidermis appeared unaffected. The ophthalmic examination was normal and other mucosal surfaces were unaffected.
Article Source

PURLs Copyright

Inside the Article

Article PDF Media

Papular Eruption Following Excessive Tanning Bed Use

Article Type
Changed
Thu, 01/10/2019 - 13:17
Display Headline
Papular Eruption Following Excessive Tanning Bed Use

The Diagnosis: Disseminated Superficial Actinic Porokeratosis

Physical examination after 7 years of tanning salon use showed a tanned white man with multiple 4- to 5-mm, discrete, round to oval, reddish brown papules on the chest, back, abdomen, arms, and legs that were rough to palpate (Figure 1), with a peripheral rim of scale seen more prominently on dermoscopy. There were no lesions on the palms or soles. A subsequent 4-mm punch biopsy was done on the right abdomen and right thigh, which showed focal thinning of the epidermis, loss of the granular layer, a discrete column of parakeratosis and a characteristic feature of a coronoid lamella in the epidermis (Figure 2). The patient received 14 narrowband UVB (NB-UVB) treatments; however, he could not continue due to transportation issues. He visited the clinic sporadically for 6 months thereafter and reportedly went to tanning salons daily. The patient was subsequently lost to follow-up.

Figure 1. Multiple 4- to 5-mm, discrete, round to oval, reddish brown papules on the abdomen (A) and leg (B) that were rough to palpate.

Disseminated porokeratosis, or disseminated superficial actinic porokeratosis (DSAP), was first described in 1967 by Chernosky and Freeman.1 It is the most common variant of porokeratosis. Other variants include Mibelli type, porokeratosis palmaris et plantaris disseminata, punctuate porokeratosis, and linear porokeratosis.2-4 Porokeratosis is inherited in an autosomal-dominant fashion, presenting in the third or fourth decades of life; however, most cases are sporadic.5 Pruritus is a common symptom and can be debilitating.5

Disseminated superficial actinic porokeratosis can be precipitated by excessive sun exposure, with a reported increase in lesions during summer months and resolution during the winter months.6 The lesions of DSAP can be experimentally induced by exposure to daily use of artificial UV sunlamps.7 Patients with psoriasis undergoing psoralen plus UVA and NB-UVB treatments also have been reported to trigger DSAP.8 A study by Neumann et al6 suggested that a combination of both UVB and UVA wavelengths may be most effective in inducing DSAP. Exposure to UVA and UVB light may explain an increased number of DSAP lesions in patients who excessively visit tanning salons, as the bulbs emit a combination of wavelengths with UVA in much greater amounts than UVB.

Our patient developed DSAP secondary to artificial UV light exposure from excessive tanning salon use. Medications (allopurinol and lisinopril) were initially thought to be etiologic agents for the eruption and also corroborated with histologic findings of a drug eruption on the initial biopsy. However, new lesions continued to develop even after cessation of medications and NB-UVB treatments. A subsequent biopsy and further history of daily tanning salon use confirmed the diagnosis of DSAP.

Biopsy revealed a discrete column of parakeratosis, a characteristic feature of a coronoid lamella in the epidermis, and moderate papillary dermis lymphocytic infiltrate (H&E, original magnification ×10)
Figure 2. Biopsy revealed a discrete column of parakeratosis, a characteristic feature of a coronoid lamella in the epidermis, and moderate papillary dermis lymphocytic infiltrate (H&E, original magnification ×10).

Therapies for this condition are limited with variable degrees of success. Cryotherapy, 5-fluorouracil cream, imiquimod cream 5%, Q-switched ruby laser, diclofenac gel 3%, and acitretin for more widespread or refractory lesions have been used with partial to complete resolution of DSAP.9

We present this case to highlight the occurrence of DSAP secondary to UV light exposure from excessive tanning salon use.

References

1. Chernosky ME, Freeman RG. Disseminated superficial actinic porokeratosis (DSAP). Arch Dermatol. 1967;96:611-624.

2. Guss SB, Osbourn RA, Lutzner MA. Porokeratosis plantaris, palmaris et disseminata: a third type of porokeratosis. Arch Dermatol. 1971;104:366-373.

3. Brown FC. Punctate keratoderma. Arch Dermatol. 1971;104:682-683.

4. Eyre WG, Carson WE. Linear porokeratosis of Mibelli. Arch Dermatol. 1972;105:426-429.

5. Anderson DE, Chernosky ME. Disseminated superficial actinic porokeratosis. Arch Dermatol. 1969;99:408-412.

6. Neumann RA, Knobler RM, Jurecka W, et al. Disseminated superficial actinic porokeratosis: experimental induction and exacerbation of skin lesions. J Am Acad Dermatol. 1989;21:1182-1188.

7. Chernosky ME, Anderson DE. Disseminated superficial actinic porokeratosis: clinical studies and experimental production of lesions. Arch Dermatol. 1969;99:401-407.

8. Allen LA, Glaser DA. Disseminated superficial actinic porokeratosis associated with topical PUVA. J Am Acad Dermatol. 2000;43:720-722.

9. Arun B, Pearson J, Chalmers R. Disseminated superficial actinic porokeratosis treated effectively with topical imiquimod 5% cream. Clin Exp Dermatol. 2011;36:509-511.

Article PDF
Author and Disclosure Information

Preethi Ramaswamy, MD; Nellie Konnikov, MD

Drs. Ramaswamy and Konnikov are from the Department of Dermatology, Boston University School of Medicine, Massachusetts. Dr. Konnikov also is from the Veterans Affairs Boston Healthcare System.

The authors report no conflict of interest.

Correspondence: Preethi Ramaswamy, MD, 609 Albany St, Boston, MA 02118 (pvenram@gmail.com).

Issue
Cutis - 94(3)
Publications
Topics
Page Number
E2-E4
Legacy Keywords
Actinic Porokeratosis, parakeratosis, Disseminated porokeratosis, Disseminated Superficial Actinic Porokeratosis, DSAP, excessive tanning,
Sections
Author and Disclosure Information

Preethi Ramaswamy, MD; Nellie Konnikov, MD

Drs. Ramaswamy and Konnikov are from the Department of Dermatology, Boston University School of Medicine, Massachusetts. Dr. Konnikov also is from the Veterans Affairs Boston Healthcare System.

The authors report no conflict of interest.

Correspondence: Preethi Ramaswamy, MD, 609 Albany St, Boston, MA 02118 (pvenram@gmail.com).

Author and Disclosure Information

Preethi Ramaswamy, MD; Nellie Konnikov, MD

Drs. Ramaswamy and Konnikov are from the Department of Dermatology, Boston University School of Medicine, Massachusetts. Dr. Konnikov also is from the Veterans Affairs Boston Healthcare System.

The authors report no conflict of interest.

Correspondence: Preethi Ramaswamy, MD, 609 Albany St, Boston, MA 02118 (pvenram@gmail.com).

Article PDF
Article PDF

The Diagnosis: Disseminated Superficial Actinic Porokeratosis

Physical examination after 7 years of tanning salon use showed a tanned white man with multiple 4- to 5-mm, discrete, round to oval, reddish brown papules on the chest, back, abdomen, arms, and legs that were rough to palpate (Figure 1), with a peripheral rim of scale seen more prominently on dermoscopy. There were no lesions on the palms or soles. A subsequent 4-mm punch biopsy was done on the right abdomen and right thigh, which showed focal thinning of the epidermis, loss of the granular layer, a discrete column of parakeratosis and a characteristic feature of a coronoid lamella in the epidermis (Figure 2). The patient received 14 narrowband UVB (NB-UVB) treatments; however, he could not continue due to transportation issues. He visited the clinic sporadically for 6 months thereafter and reportedly went to tanning salons daily. The patient was subsequently lost to follow-up.

Figure 1. Multiple 4- to 5-mm, discrete, round to oval, reddish brown papules on the abdomen (A) and leg (B) that were rough to palpate.

Disseminated porokeratosis, or disseminated superficial actinic porokeratosis (DSAP), was first described in 1967 by Chernosky and Freeman.1 It is the most common variant of porokeratosis. Other variants include Mibelli type, porokeratosis palmaris et plantaris disseminata, punctuate porokeratosis, and linear porokeratosis.2-4 Porokeratosis is inherited in an autosomal-dominant fashion, presenting in the third or fourth decades of life; however, most cases are sporadic.5 Pruritus is a common symptom and can be debilitating.5

Disseminated superficial actinic porokeratosis can be precipitated by excessive sun exposure, with a reported increase in lesions during summer months and resolution during the winter months.6 The lesions of DSAP can be experimentally induced by exposure to daily use of artificial UV sunlamps.7 Patients with psoriasis undergoing psoralen plus UVA and NB-UVB treatments also have been reported to trigger DSAP.8 A study by Neumann et al6 suggested that a combination of both UVB and UVA wavelengths may be most effective in inducing DSAP. Exposure to UVA and UVB light may explain an increased number of DSAP lesions in patients who excessively visit tanning salons, as the bulbs emit a combination of wavelengths with UVA in much greater amounts than UVB.

Our patient developed DSAP secondary to artificial UV light exposure from excessive tanning salon use. Medications (allopurinol and lisinopril) were initially thought to be etiologic agents for the eruption and also corroborated with histologic findings of a drug eruption on the initial biopsy. However, new lesions continued to develop even after cessation of medications and NB-UVB treatments. A subsequent biopsy and further history of daily tanning salon use confirmed the diagnosis of DSAP.

Biopsy revealed a discrete column of parakeratosis, a characteristic feature of a coronoid lamella in the epidermis, and moderate papillary dermis lymphocytic infiltrate (H&E, original magnification ×10)
Figure 2. Biopsy revealed a discrete column of parakeratosis, a characteristic feature of a coronoid lamella in the epidermis, and moderate papillary dermis lymphocytic infiltrate (H&E, original magnification ×10).

Therapies for this condition are limited with variable degrees of success. Cryotherapy, 5-fluorouracil cream, imiquimod cream 5%, Q-switched ruby laser, diclofenac gel 3%, and acitretin for more widespread or refractory lesions have been used with partial to complete resolution of DSAP.9

We present this case to highlight the occurrence of DSAP secondary to UV light exposure from excessive tanning salon use.

The Diagnosis: Disseminated Superficial Actinic Porokeratosis

Physical examination after 7 years of tanning salon use showed a tanned white man with multiple 4- to 5-mm, discrete, round to oval, reddish brown papules on the chest, back, abdomen, arms, and legs that were rough to palpate (Figure 1), with a peripheral rim of scale seen more prominently on dermoscopy. There were no lesions on the palms or soles. A subsequent 4-mm punch biopsy was done on the right abdomen and right thigh, which showed focal thinning of the epidermis, loss of the granular layer, a discrete column of parakeratosis and a characteristic feature of a coronoid lamella in the epidermis (Figure 2). The patient received 14 narrowband UVB (NB-UVB) treatments; however, he could not continue due to transportation issues. He visited the clinic sporadically for 6 months thereafter and reportedly went to tanning salons daily. The patient was subsequently lost to follow-up.

Figure 1. Multiple 4- to 5-mm, discrete, round to oval, reddish brown papules on the abdomen (A) and leg (B) that were rough to palpate.

Disseminated porokeratosis, or disseminated superficial actinic porokeratosis (DSAP), was first described in 1967 by Chernosky and Freeman.1 It is the most common variant of porokeratosis. Other variants include Mibelli type, porokeratosis palmaris et plantaris disseminata, punctuate porokeratosis, and linear porokeratosis.2-4 Porokeratosis is inherited in an autosomal-dominant fashion, presenting in the third or fourth decades of life; however, most cases are sporadic.5 Pruritus is a common symptom and can be debilitating.5

Disseminated superficial actinic porokeratosis can be precipitated by excessive sun exposure, with a reported increase in lesions during summer months and resolution during the winter months.6 The lesions of DSAP can be experimentally induced by exposure to daily use of artificial UV sunlamps.7 Patients with psoriasis undergoing psoralen plus UVA and NB-UVB treatments also have been reported to trigger DSAP.8 A study by Neumann et al6 suggested that a combination of both UVB and UVA wavelengths may be most effective in inducing DSAP. Exposure to UVA and UVB light may explain an increased number of DSAP lesions in patients who excessively visit tanning salons, as the bulbs emit a combination of wavelengths with UVA in much greater amounts than UVB.

Our patient developed DSAP secondary to artificial UV light exposure from excessive tanning salon use. Medications (allopurinol and lisinopril) were initially thought to be etiologic agents for the eruption and also corroborated with histologic findings of a drug eruption on the initial biopsy. However, new lesions continued to develop even after cessation of medications and NB-UVB treatments. A subsequent biopsy and further history of daily tanning salon use confirmed the diagnosis of DSAP.

Biopsy revealed a discrete column of parakeratosis, a characteristic feature of a coronoid lamella in the epidermis, and moderate papillary dermis lymphocytic infiltrate (H&E, original magnification ×10)
Figure 2. Biopsy revealed a discrete column of parakeratosis, a characteristic feature of a coronoid lamella in the epidermis, and moderate papillary dermis lymphocytic infiltrate (H&E, original magnification ×10).

Therapies for this condition are limited with variable degrees of success. Cryotherapy, 5-fluorouracil cream, imiquimod cream 5%, Q-switched ruby laser, diclofenac gel 3%, and acitretin for more widespread or refractory lesions have been used with partial to complete resolution of DSAP.9

We present this case to highlight the occurrence of DSAP secondary to UV light exposure from excessive tanning salon use.

References

1. Chernosky ME, Freeman RG. Disseminated superficial actinic porokeratosis (DSAP). Arch Dermatol. 1967;96:611-624.

2. Guss SB, Osbourn RA, Lutzner MA. Porokeratosis plantaris, palmaris et disseminata: a third type of porokeratosis. Arch Dermatol. 1971;104:366-373.

3. Brown FC. Punctate keratoderma. Arch Dermatol. 1971;104:682-683.

4. Eyre WG, Carson WE. Linear porokeratosis of Mibelli. Arch Dermatol. 1972;105:426-429.

5. Anderson DE, Chernosky ME. Disseminated superficial actinic porokeratosis. Arch Dermatol. 1969;99:408-412.

6. Neumann RA, Knobler RM, Jurecka W, et al. Disseminated superficial actinic porokeratosis: experimental induction and exacerbation of skin lesions. J Am Acad Dermatol. 1989;21:1182-1188.

7. Chernosky ME, Anderson DE. Disseminated superficial actinic porokeratosis: clinical studies and experimental production of lesions. Arch Dermatol. 1969;99:401-407.

8. Allen LA, Glaser DA. Disseminated superficial actinic porokeratosis associated with topical PUVA. J Am Acad Dermatol. 2000;43:720-722.

9. Arun B, Pearson J, Chalmers R. Disseminated superficial actinic porokeratosis treated effectively with topical imiquimod 5% cream. Clin Exp Dermatol. 2011;36:509-511.

References

1. Chernosky ME, Freeman RG. Disseminated superficial actinic porokeratosis (DSAP). Arch Dermatol. 1967;96:611-624.

2. Guss SB, Osbourn RA, Lutzner MA. Porokeratosis plantaris, palmaris et disseminata: a third type of porokeratosis. Arch Dermatol. 1971;104:366-373.

3. Brown FC. Punctate keratoderma. Arch Dermatol. 1971;104:682-683.

4. Eyre WG, Carson WE. Linear porokeratosis of Mibelli. Arch Dermatol. 1972;105:426-429.

5. Anderson DE, Chernosky ME. Disseminated superficial actinic porokeratosis. Arch Dermatol. 1969;99:408-412.

6. Neumann RA, Knobler RM, Jurecka W, et al. Disseminated superficial actinic porokeratosis: experimental induction and exacerbation of skin lesions. J Am Acad Dermatol. 1989;21:1182-1188.

7. Chernosky ME, Anderson DE. Disseminated superficial actinic porokeratosis: clinical studies and experimental production of lesions. Arch Dermatol. 1969;99:401-407.

8. Allen LA, Glaser DA. Disseminated superficial actinic porokeratosis associated with topical PUVA. J Am Acad Dermatol. 2000;43:720-722.

9. Arun B, Pearson J, Chalmers R. Disseminated superficial actinic porokeratosis treated effectively with topical imiquimod 5% cream. Clin Exp Dermatol. 2011;36:509-511.

Issue
Cutis - 94(3)
Issue
Cutis - 94(3)
Page Number
E2-E4
Page Number
E2-E4
Publications
Publications
Topics
Article Type
Display Headline
Papular Eruption Following Excessive Tanning Bed Use
Display Headline
Papular Eruption Following Excessive Tanning Bed Use
Legacy Keywords
Actinic Porokeratosis, parakeratosis, Disseminated porokeratosis, Disseminated Superficial Actinic Porokeratosis, DSAP, excessive tanning,
Legacy Keywords
Actinic Porokeratosis, parakeratosis, Disseminated porokeratosis, Disseminated Superficial Actinic Porokeratosis, DSAP, excessive tanning,
Sections
Questionnaire Body
Papular Eruption Following Excessive Tanning Bed Use
Papular Eruption Following Excessive Tanning Bed Use

A 78-year-old man with Fitzpatrick skin type III presented to the dermatology department for evaluation of a pruritic, erythematous, papular eruption on the chest, back, abdomen, arms, and legs of 5 years’ duration. His medications include clonazepam, lisinopril, allopurinol, omeprazole, tramadol, and mirtazapine. The lesions did not respond to topical corticosteroids; however, the pruritus improved with narrowband UVB (NB-UVB) treatments. Review of systems did not reveal any abnormalities. The patient’s medical history included gout, hypertension, anxiety, esophageal stricture, and emphysema. He reported a history of tanning salon use at least 3 times weekly for 7 years. After initial consultation, the patient was treated with clobetasol propionate cream 0.05% twice daily and hydroxyzine 10 mg 3 times daily. Following 1 month of treatment, the eruption did not improve. A 4-mm punch biopsy of the left upper arm revealed a dense infiltrate in the upper dermis with prominent parakeratosis, lymphocytes, and numerous eosinophils, suggestive of a drug eruption. As a result, allopurinol was discontinued as a causative agent; however, the eruption presented prior to taking allopurinol. Because the patient experienced intense pruritus, he was started on NB-UVB treatments. After 14 treatments of NB-UVB 3 times weekly, the patient noticed some improvement with respect to pruritus, but the lesions did not resolve. A complete blood cell count indicated 7.6% eosinophils (reference range, 0%–5%). Liver function tests, complete metabolic profile, and renal function were within reference range. Lisinopril was then discontinued as a likely culprit for persistent drug eruption; however, new lesions continued to develop.
Article Source

PURLs Copyright

Inside the Article

Article PDF Media

“Something Abnormal” on a Chest X-ray

Article Type
Changed
Mon, 07/09/2018 - 10:49
Display Headline
“Something Abnormal” on a Chest X-ray

ANSWER
The radiograph demonstrates a fairly large (4 x 6 cm) right paratracheal mass of unclear etiology. This type of finding warrants further evaluation with contrasted CT.

Fortunately for this patient, a subsequent study demonstrated a slightly enlarged thyroid gland. This correlated with the radiographic
finding.       

References

Article PDF
Author and Disclosure Information

Nandan R. Hichkad, PA-C, MMSc, practices at the Georgia Neurosurgical Institute in Macon.

Issue
Clinician Reviews - 24(9)
Publications
Topics
Page Number
14,49
Legacy Keywords
paratracheal, radiology, review, preoperative, preop, hematoma, subdural, paratracheal mass,
Sections
Author and Disclosure Information

Nandan R. Hichkad, PA-C, MMSc, practices at the Georgia Neurosurgical Institute in Macon.

Author and Disclosure Information

Nandan R. Hichkad, PA-C, MMSc, practices at the Georgia Neurosurgical Institute in Macon.

Article PDF
Article PDF

ANSWER
The radiograph demonstrates a fairly large (4 x 6 cm) right paratracheal mass of unclear etiology. This type of finding warrants further evaluation with contrasted CT.

Fortunately for this patient, a subsequent study demonstrated a slightly enlarged thyroid gland. This correlated with the radiographic
finding.       

ANSWER
The radiograph demonstrates a fairly large (4 x 6 cm) right paratracheal mass of unclear etiology. This type of finding warrants further evaluation with contrasted CT.

Fortunately for this patient, a subsequent study demonstrated a slightly enlarged thyroid gland. This correlated with the radiographic
finding.       

References

References

Issue
Clinician Reviews - 24(9)
Issue
Clinician Reviews - 24(9)
Page Number
14,49
Page Number
14,49
Publications
Publications
Topics
Article Type
Display Headline
“Something Abnormal” on a Chest X-ray
Display Headline
“Something Abnormal” on a Chest X-ray
Legacy Keywords
paratracheal, radiology, review, preoperative, preop, hematoma, subdural, paratracheal mass,
Legacy Keywords
paratracheal, radiology, review, preoperative, preop, hematoma, subdural, paratracheal mass,
Sections
Questionnaire Body
Radiograph of preoperative patient after fall.
Chest radiograph of preop patient with acute left subdural hematoma after ground level fall.

You are doing preoperative orders on a patient scheduled for surgery tomorrow morning. The patient is a 75-year-old woman who was admitted with an acute left subdural hematoma after sustaining a ground-level fall. Her medical history is significant for hypertension and diabetes. Social history is unremarkable. She is neurologically intact except for occasional confusion and aphasia. She moves all her extremities well. As you review her lab results, one of the nurses mentions that the radiology department called about “something abnormal” on the patient’s chest radiograph. You pull up the patient’s portable chest radiograph on the computer to review. What is your impression?
Article Source

PURLs Copyright

Inside the Article

Article PDF Media

Itchy Lesion Heralds Pervasive Problem

Article Type
Changed
Tue, 12/13/2016 - 12:08
Display Headline
Itchy Lesion Heralds Pervasive Problem

ANSWER
The correct answer is pityriasis rosea (choice “b”), a common and very distinctive eruption related to human herpesvirus 6 and 7.

Allergic reaction to methotrexate (choice “a”), while far from unknown, does not resemble pityriasis rosea. It also would not be limited to such a relatively small area.

Pityriasis rosea is often designated as “fungal infection” (choice “c”) by the uninitiated. However, the lesions of dermatophytosis would be round, with a leading scaly edge, and unlikely to be found in this distribution.

Secondary syphilis (choice “d”) is a major item in the pityriasis rosea differential, but it almost always involves the palms and soles and the lesions would be round (not oval) scaly brown papules. Furthermore, assuming we have an honest patient, we’re also missing a source for sexually transmitted infection.

DISCUSSION
One could hardly ask for a more classic case of pityriasis rosea (PR), which primarily affects patients ages 14 to 40. Alas, that being said, one cannot depend on seeing all these clues in every PR patient.

For example, the herald patch (also known as the mother patch) is missing in at least half of cases. In others, the lesions are smaller, sparser, and more papular (especially in young black patients). The condition may even be confined to intertriginous areas (eg, the groin and/or axillae); this is known as inverse PR.

While salmon-colored scaly lesions are considered a classic presentation, PR can present with darker ovoid macules that have minimal scale and, rarely, become bullous. Involvement above the neck is rare.

What is consistent and dependable among signs of PR is the centripetal scale, seen even in the smallest lesions. This scale is so fine that the old dermatology texts called it “cigarette paper” scale or “scurf.”

After decades of speculation, researchers finally provided strong evidence of the probable cause  of PR: replication of human herpesvirus 6 and 7, present in mono-
nuclear cells of lesional skin. Though universally acquired in childhood, these viruses are thought to remain latent until reactivated, leading to viremia.

Itching can be moderately severe in a minority of cases. Most patients, such as this one, are not bothered much by the condition once they understand its self-limited nature. They usually are not happy, however, to learn that it could persist for nine weeks or more, whether treated or not.

UV light exposure can be helpful in hastening PR’s departure, and topical corticosteroids (class III or IV; eg, triamcinolone 0.1% cream) can help control the itching. Neither oral nor topical antihistamines will help, since PR is not a histamine-mediated problem.

If the diagnosis is in doubt, a punch biopsy could at least rule out the more serious items in the differential, which include syphilis, drug rash, and psoriasis. In cases in which fungal origin is a possibility, a quick KOH prep will settle the issue. However, it must be remembered that one doesn’t just “get” a fungal infection. There has to be a source (animal, child), and that source is usually identified with minimal history taking.

Article PDF
Author and Disclosure Information

 

Joe R. Monroe, MPAS, PA, ­practices at Dawkins ­Dermatology Clinic in Oklahoma City. He is also the founder of the Society of ­Dermatology ­Physician ­Assistants.

Issue
Clinician Reviews - 24(9)
Publications
Topics
Page Number
11-12
Legacy Keywords
dermatology, dermadiagnosis, neck, chest, rash, allergic reaction, methotrexate, Pityriasis rosea, fungal infection, Secondary syphilis
Sections
Author and Disclosure Information

 

Joe R. Monroe, MPAS, PA, ­practices at Dawkins ­Dermatology Clinic in Oklahoma City. He is also the founder of the Society of ­Dermatology ­Physician ­Assistants.

Author and Disclosure Information

 

Joe R. Monroe, MPAS, PA, ­practices at Dawkins ­Dermatology Clinic in Oklahoma City. He is also the founder of the Society of ­Dermatology ­Physician ­Assistants.

Article PDF
Article PDF

ANSWER
The correct answer is pityriasis rosea (choice “b”), a common and very distinctive eruption related to human herpesvirus 6 and 7.

Allergic reaction to methotrexate (choice “a”), while far from unknown, does not resemble pityriasis rosea. It also would not be limited to such a relatively small area.

Pityriasis rosea is often designated as “fungal infection” (choice “c”) by the uninitiated. However, the lesions of dermatophytosis would be round, with a leading scaly edge, and unlikely to be found in this distribution.

Secondary syphilis (choice “d”) is a major item in the pityriasis rosea differential, but it almost always involves the palms and soles and the lesions would be round (not oval) scaly brown papules. Furthermore, assuming we have an honest patient, we’re also missing a source for sexually transmitted infection.

DISCUSSION
One could hardly ask for a more classic case of pityriasis rosea (PR), which primarily affects patients ages 14 to 40. Alas, that being said, one cannot depend on seeing all these clues in every PR patient.

For example, the herald patch (also known as the mother patch) is missing in at least half of cases. In others, the lesions are smaller, sparser, and more papular (especially in young black patients). The condition may even be confined to intertriginous areas (eg, the groin and/or axillae); this is known as inverse PR.

While salmon-colored scaly lesions are considered a classic presentation, PR can present with darker ovoid macules that have minimal scale and, rarely, become bullous. Involvement above the neck is rare.

What is consistent and dependable among signs of PR is the centripetal scale, seen even in the smallest lesions. This scale is so fine that the old dermatology texts called it “cigarette paper” scale or “scurf.”

After decades of speculation, researchers finally provided strong evidence of the probable cause  of PR: replication of human herpesvirus 6 and 7, present in mono-
nuclear cells of lesional skin. Though universally acquired in childhood, these viruses are thought to remain latent until reactivated, leading to viremia.

Itching can be moderately severe in a minority of cases. Most patients, such as this one, are not bothered much by the condition once they understand its self-limited nature. They usually are not happy, however, to learn that it could persist for nine weeks or more, whether treated or not.

UV light exposure can be helpful in hastening PR’s departure, and topical corticosteroids (class III or IV; eg, triamcinolone 0.1% cream) can help control the itching. Neither oral nor topical antihistamines will help, since PR is not a histamine-mediated problem.

If the diagnosis is in doubt, a punch biopsy could at least rule out the more serious items in the differential, which include syphilis, drug rash, and psoriasis. In cases in which fungal origin is a possibility, a quick KOH prep will settle the issue. However, it must be remembered that one doesn’t just “get” a fungal infection. There has to be a source (animal, child), and that source is usually identified with minimal history taking.

ANSWER
The correct answer is pityriasis rosea (choice “b”), a common and very distinctive eruption related to human herpesvirus 6 and 7.

Allergic reaction to methotrexate (choice “a”), while far from unknown, does not resemble pityriasis rosea. It also would not be limited to such a relatively small area.

Pityriasis rosea is often designated as “fungal infection” (choice “c”) by the uninitiated. However, the lesions of dermatophytosis would be round, with a leading scaly edge, and unlikely to be found in this distribution.

Secondary syphilis (choice “d”) is a major item in the pityriasis rosea differential, but it almost always involves the palms and soles and the lesions would be round (not oval) scaly brown papules. Furthermore, assuming we have an honest patient, we’re also missing a source for sexually transmitted infection.

DISCUSSION
One could hardly ask for a more classic case of pityriasis rosea (PR), which primarily affects patients ages 14 to 40. Alas, that being said, one cannot depend on seeing all these clues in every PR patient.

For example, the herald patch (also known as the mother patch) is missing in at least half of cases. In others, the lesions are smaller, sparser, and more papular (especially in young black patients). The condition may even be confined to intertriginous areas (eg, the groin and/or axillae); this is known as inverse PR.

While salmon-colored scaly lesions are considered a classic presentation, PR can present with darker ovoid macules that have minimal scale and, rarely, become bullous. Involvement above the neck is rare.

What is consistent and dependable among signs of PR is the centripetal scale, seen even in the smallest lesions. This scale is so fine that the old dermatology texts called it “cigarette paper” scale or “scurf.”

After decades of speculation, researchers finally provided strong evidence of the probable cause  of PR: replication of human herpesvirus 6 and 7, present in mono-
nuclear cells of lesional skin. Though universally acquired in childhood, these viruses are thought to remain latent until reactivated, leading to viremia.

Itching can be moderately severe in a minority of cases. Most patients, such as this one, are not bothered much by the condition once they understand its self-limited nature. They usually are not happy, however, to learn that it could persist for nine weeks or more, whether treated or not.

UV light exposure can be helpful in hastening PR’s departure, and topical corticosteroids (class III or IV; eg, triamcinolone 0.1% cream) can help control the itching. Neither oral nor topical antihistamines will help, since PR is not a histamine-mediated problem.

If the diagnosis is in doubt, a punch biopsy could at least rule out the more serious items in the differential, which include syphilis, drug rash, and psoriasis. In cases in which fungal origin is a possibility, a quick KOH prep will settle the issue. However, it must be remembered that one doesn’t just “get” a fungal infection. There has to be a source (animal, child), and that source is usually identified with minimal history taking.

Issue
Clinician Reviews - 24(9)
Issue
Clinician Reviews - 24(9)
Page Number
11-12
Page Number
11-12
Publications
Publications
Topics
Article Type
Display Headline
Itchy Lesion Heralds Pervasive Problem
Display Headline
Itchy Lesion Heralds Pervasive Problem
Legacy Keywords
dermatology, dermadiagnosis, neck, chest, rash, allergic reaction, methotrexate, Pityriasis rosea, fungal infection, Secondary syphilis
Legacy Keywords
dermatology, dermadiagnosis, neck, chest, rash, allergic reaction, methotrexate, Pityriasis rosea, fungal infection, Secondary syphilis
Sections
Questionnaire Body

Patient with rash on chest and neck.
This itchy rash started on his back and spread to his chest and neck.

 

 

Two weeks ago, an itchy rash appeared on a man’s back before spreading to his chest and neck. He has never experienced anything like it before, and no one else in his household is similarly affected. He denies night sweats, fever, and malaise but reports that he was recently diagnosed with rheumatoid arthritis. His rheumatologist started him on methotrexate (12.5 mg/wk) after extensive labwork (complete blood count, complete metabolic panel, and hepatitis profile) was performed. He denies any history of high-risk sexual behavior or exposure, exposure to animals or children, or history of foreign travel. The patient, who appears well, is afebrile and in no distress. The original lesion, on his upper left back, is distinctly pinkish brown and round, with an odd fine scale around its inner rim, and measures about 3 cm in diameter. Elsewhere, examination reveals about 15 more lesions. All are oval but similarly pinkish brown, averaging about 2 cm in their long axis. These smaller lesions form a necklace-like configuration, paralleling the natural skin lines of the neck. Each lesion has central scaling identical to that of the original back lesion. Examination of the patient’s palms and soles fails to reveal any cutaneous abnormalities. Likewise, examination of the oral cavity is normal. No palpable nodes are felt around the neck, in the axillae, or in the groin.

 

Disallow All Ads
Article PDF Media